Evidence Final for 2Ls

  1. Clyde is charged with robbing a bank, but in the federal court case alleges that he was vacationing 5 states away in Wisconsin when the robbery occurreD) Clyde calls Bonnie as an alibi to testify that she was traveling on vacation with Clyde at the time in question. When questioned where she was during the robbery and with whom, Bonnie says, "Yes, I definitely know that I was at the Grizzard Motel the weekend of the robbery, because that is the same weekend as my birthday and I was there to celebrate. The thing is, there's no way I was there with a guy as ugly as Clyde." Clyde's attorney seeks to introduce evidence that Bonnie was at the Grizzard Motel with Clyde. Clyde had testified to that fact in the preliminary hearing. The prosecution objects. The question is:A) Proper, if the judge determines that Bonnie is a hostile witness.B) Proper, as it is simply refreshing the witness's recollection.C) Improper, as it is a leading question on direct.D) Improper, since Clyde's attorney cannot impeach her own witness.
    The federal rules state that an attorney may impeach her own witness, even on direct, if the judge has found the witness to be a hostile witness. In light of this, Answers C and D are clearly incorrect.B is wrong as Clyde's attorney is not seeking to refresh Bonnie's memory, but to admit testimony from the preliminary hearing in order to impeach her.
  2. Dumaine owns a bakery, and is charged with murdering Hansel, his employee, by putting him in the baking oven with some muffins. The prosecution seeks to introduce the testimony of Oliver, another employee of Dumaine. Oliver will testify that as he went to place an order for more flour one day, he accidentally pushed an already lit extension on the phone and overheard an unknown voice say to Dumaine, "Hey Dumaine, I hear that Hansel is going to testify about all the illegal prescription drugs that you have been selling out of the bakery. What are you going to do about him?" Dumaine's attorney objects to the introduction of Oliver's testimony on the basis that it is irrelevant. How should the judge rule? A) Sustain the objection.B) Overrule the objection, as this testimony is being introduced to show Dumaine's state of mind and is, therefore, within a hearsay exception.C) Overrule the objection, as this hearsay falls within the business record exception, since the attempted call was made as a part of work during business hours.D) Overrule the objection if, and only if, the evidence is offered to prove motive.
    Recall that an out-of-court statement offered for the purpose of proving knowledge on the part of the listener is not deemed objectionable hearsay. Furthermore, the evidence may be admitted as an admission by silence on the part of Dumaine. In light of the correct answer, A is clearly wrong.B is wrong. The state of mind exception only applies if an out-of-court statement is introduced in an attempt to prove the declarant's state of minD) Here, that is clearly not the case, making this exception inapplicable.C is wrong. The business record exception does not apply to phone calls made during work hours, only to work product that is generated in the course of business (i.e., charts or documents).
  3. Thornton is walking to class when he sees a red SUV blow through a red light, hit an elderly pedestrian knocking him unconscious, and continue on through the intersection. Thornton calls the police and gives a detailed description of what he saw. The police capture Michaleen. They charge him with numerous counts, including hit-and-run and battery of the elderly pedestrian. Thornton testifies at trial, but Michaleen manages to get acquitteD) The elderly pedestrian then sues in a civil suit against Michaleen for damages relating to his injuries. Just before the civil suit, as luck would have it, Thornton is run over by a runaway truck full of hogs at the same exact intersection. The truck is driven by Wayne. An ambulance rushes Thornton to the hospital who with his last breath calls out, "Michaleen ran over that elderly man. I will never forget his face. He is guilty! Guilty I tell your Thornton then dies. The elderly man's attorney calls the EMT from the ambulance who was looking after Thornton. She is asked to testify as to Thornton's last words. The EMT's testimony is:A) Admissible, because we are dealing with a dying declaration.B) Admissible, because we are dealing with an excited utterance.C) Inadmissible, because of the doctor/patient privilege.D) Inadmissible, because it is hearsay not within any exception.
    Here, we have classic hearsay, namely, an out-of-court declaration offered for its truth, and there is no applicable exception that may apply. In light of this, D is correct.A is wrong as a dying declaration requires that the declaration concern the immediate circumstance causing Thornton's deadly injury. (Thornton stating that the truck that hit him ran a red light, for example.)B is also wrong. Excited utterance only applies to a situation where the statement is given immediately after seeing the startling event. Here, too much time has clearly passeD)C is incorrect. First, it is unclear whether the EMT shares doctor/patient privilege with Thornton. Even if she did, the privilege would only extend to info given in the course of Thornton's treatment. Thornton's statement concerning the elderly man's accident is well beyond the course of his treatment and, therefore, unprotected.
  4. Yuliana and Jay help Candace unload boxes of her books onto a loading dock in front of her new apartment. Candace loves books and is using a forklift to carry her heavy collection. As she is driving the forklift, she collides with Kendra in her car. As they collide, the load of books tumbles to the ground and falls on Eve, who was walking in the roaD) Eve is hurt. A shopkeeper across the street sees all this happen and calls the police who arrive to take statements. Kendra sues Candace for negligence, claiming that Candace should not have been driving a forklift in the roaD) Kendra also asserts that she was driving in observance of all the traffic laws. Candace claims that it was Kendra who really swerved into her and the forklift, because Kendra was distracted on her cell phone. All of the witnesses are available to testify at trial. Assume that Kendra is suing Candace for damages relating to injuries sustaineD) Kendra's attorney seeks to introduce testimony by Kendra to her husband 3 days after the accident that states, "My neck is killing me! I must have pulled a muscle in it during the accident." The judge should rule this testimony:A) Inadmissible; it is hearsay.B) Inadmissible; Kendra has no qualification to give testimony relating to her medical condition.C) Admissible, if Kendra put evidence forward showing that the medical diagnosis is in keeping with her alleged sprain.D) Admissible to show the pain that was suffered by Kendra.
    The testimony relating to the showing of pain in a witness falls within the hearsay exception of a present physical impression. In light of this, Answer A is incorrect as we are within a hearsay exception.B is wrong. The testimony is not offered to show that Kendra suffered a sprained muscle, which would require an expert opinion, but rather to show that Kendra was in pain.C is wrong. This statement can be admitted to show Kendra's current physical condition, even without a showing of medical evidence to support it.
  5. Minka is charged with bank robbery. During her prosecution, it is well established that the robber left the bank wearing a dog mask and took off in a getaway car. Bill, Gill and Dan all witness the getaway. Bill is ready to testify that when Minka jumped into the car someone yelled "Get the plate number.!" and Bill replied, 7 got Itl It is 1234567F' Bill's testimony is admitted into the case. Gill then wishes to testify as a witness for the defense that he was present at the scene and does not remember hearing Bill say anything. This testimony is:A) Inadmissible, since it is irrelevant.B) Proper impeachment of Bill.C) Improper impeachment of Bill, since it relates to a collateral matter.D) Improper impeachment, because Gill only states that he does not remember Bill's comment. He does not expressly contradict Bill's testimony.
    By definition, impeachment is the casting of an adverse reflection on the truth or veracity of a witness. Impeachment can come during cross-examination, or can occur by putting another witness on the stand to testify counter to the first witness's testimony (as is happening here).A is wrong since credibility of a witness is always an issue. Furthermore, the proposed evidence offered by the second witness is clearly relevant, as it is addresses a matter at issue in the case.C is wrong. We are not dealing with a collateral matter.Note: Impeachment based on a collateral matter is generally not permissible, wherein collateral matters are those that are not relevant.Here, however, we are clearly dealing with a matter that is of great relevance.D is wrong since impeachment evidence does not need to completely contradict a prior statement. It only needs to cast doubt as to the prior witness's truth or credibility.
  6. Cami sues Bruce for frauD) She seeks damages following the purchase of a charter fishing business from Bruce. Cami claims that she relied on Bruce's bookkeeping as to the value of the business based on assets and inventory (frozen bait). Cami also claims that the bookkeeping turned out to be fraudulent. At trial, Cami's attorney introduces the written contract, without objection from defense, which reads, "HolyMackeral is a solvent company with inventory and assets valued at $1 million." The contract further includes some boilerplate terms like "no other additional or contradictory representations have been made by the seller to the buyer beyond those written above, etC).." Note that Bruce is a lawyer, while Cami is not. Cami's attorney seeks to call Bruce's ex-wife, claiming that Bruce told his wife all about how he pulled a fast one on Cami. Bruce objects, claiming spousal privilege. Can his ex-wife testify?A) Yes, she can be called and can testify.B) Yes, she can be called and must testify since the privilege alleged only applies to criminal cases.C) No, she cannot be directed to testify if Bruce objects and they were still married at the time he spoke to her about the deal with Cami.D) No, she cannot be called as a witness at all if Bruce objects, regardless of when they were divorced.
    Bruce's privilege to prevent his ex-wife from testifying is applicable if he confided in her while they were marrieD) Note that both spouses have the privilege to not disclose and either spouse can also prevent the other person from disclosing. Further note that this only applies if the communication was made while they were still married, and in reliance upon that confidential married relationship. Divorce does not serve to terminate this privilege retroactively.A is wrong for failing to address Bruce's privilege to prevent his ex-wife from testifying.B is incorrect. Spousal privilege does, in fact, extend to civil cases. Note that when dealing with criminal cases the concept at hand is spousal immunity, wherein a married person may not be compelled to testify against his or her spouse, and vice versA) Spousal immunity, however, stops upon divorce or annulment.D is incorrect for stating an answer that is too broaD) Bruce cannot prevent his ex-wife from being called as a witness at trial. All that Bruce can do is prevent his ex-wife from testifying relating to matters that he revealed during their marriage.
  7. Strauss sues Morrow and Morrow's employer based on a personal injury claim in which Strauss is hit on the head by a bucket of paint that was dropped by Morrow while she was painting the Golden Strait Bridge. To prove that the paint was dropped by Morrow, Strauss's attorney introduces the now empty and dented paint can. The can has a sticker attached stating the paint color: "Strait Bridge golD)" Strauss's attorney also introduces evidence that the same brand of "Strait Bridge gold" paint was sold to Morrow and her employer for use in painting the bridge. Morrow's attorney seeks to introduce evidence that Strauss assumed the risk of walking. Morrow's attorney also wishes to introduce the statement of Gertrude, another painting employee willing to testify, who said to Strauss just before the can hit him, "Hey mister, if you start walking across this side of the bridge without a hard hat, there is a good chance you might not make it to the other end in one piece" Which of the following best characterizes this testimony?A) The evidence is not hearsay because the declarant is testifying as a witness at the hearing.B) The evidence is hearsay, but it should be admitted as part of the res gestae exception.C) The evidence is hearsay, but it should be admitted under the present state of mind exception to the hearsay rule.D) The evidence is not hearsay because the statement is not offered for its truth.
    Here, Gertrude's statement is not being offered for truth, but rather to show the effect on Strauss, regarding his assumption of risk. Thus, in a negligence case, where knowledge of a danger is at issue, a statement like this can be admitted for the very limited reason of showing knowledge or notice on the part of a listener. The statement is not hearsay because it is not offered to prove that it was, in fact, dangerous for Strauss to start walking across the bridge without a hard hat.A is the right outcome for the wrong reason, as it states that this is not hearsay simply because Gertrude is testifying. This is meaningless as any out-of-court statement offered for its truth is hearsay in most jurisdictions, regardless of whether the declarant is testifying. The fine point you MUST pick up on here is NOT the fact that the declarant (Gertrude) herself is testifying, but rather the fact that the statement is not being offered for its truth.B is wrong for stating that we are dealing with hearsay, which is wrong since Gertrude's statement is not being offered for its truth. The category of res gestae exceptions to the hearsay rule includes the following: 1) present state of mind; 2) excited utterances; 3) present sense impression; and 4) declarations of physical condition. Gertrude's testimony fails to fall under any of these.C incorrectly characterizes the testimony as hearsay. Furthermore, even if the statement washearsay, the present state of mind exception is not applicable. Recall that the present state of mind exception allows for admissibility of evidence regarding a declarant's existing state of mind at the time of controversy only when the declarant's state of mind is directly in issue and material to the controversy. Here, Gertrude's state of mind at the time of Strauss's injury has no relevance to the matter at hanD)
  8. Sylvester and Rea collide with each other in their fancy sports cars. Rea is indicted for driving drunk, which has a maximum sentence of 2 years in jail. Blathe, who witnessed the entire accident, testifies in front of the grand jury. Rea pleads guilty and she is fined $500. After completion of the criminal proceedings, Sylvester sues Rea in a negligence suit, seeking personal injury damages. At the negligence trial, neither attorney can find Blathe, but Carla steps up to testify with testimony similar to Blathe's from the grand jury trial. Rea's attorney then offers a properly authenticated transcript of Blathe's grand jury evidence, without giving any notice of this. The transcript is:A) Admissible, in light of the former testimony exception to the rule against hearsay.B) Admissible nonhearsayC) Admissible to rehabilitate the testimony of Carla, assuming that Carla was impeached with a charge of fabrication or motive against Sylvester.D) Inadmissible hearsay
    Here, we clearly have inadmissible hearsay as the transcript of Blathe's grand jury testimony is being offered to prove the truth of the matter asserted therein and Blathe is unavailable for cross-examination. Furthermore, this testimony does not fall within a hearsay exception.A is wrong for stating an inapplicable hearsay exception. The former testimony exception only applies to testimony of an unavailable witness, which was given at another hearing and taken in accordance with law, if there is a sufficient similarity of parties and issues, and both parties in the prior hearing had a chance to develop cross-examination plans of attack. Recall that grand jury proceedings do not afford an opportunity for cross-examination, making this exception inapplicable.B is incorrect because Blathe's testimony is clearly hearsay and does not fall within any exception.C is incorrect. Rehabilitation of a witness (Carla) can only be accomplished by using prior consistent testimony statements of that witness, not statements of another witness that would have been consistent. An introduced statement from Carla could be admitted both to bolster her testimony and as substantive evidence of the truth of its contents.
  9. Samir sues Kylie for breach of contract regarding the purchase of 10 boxes of new Cocoon digital music players. Samir alleges that Kylie failed to pay as stated in their contract. Kylie's attorney defends this allegation by introducing the fact that Samir only sent a shipment of 6 boxes (not the intended 10) which she rejecteD) Samir's attorney introduces a bill of lading stating that 10 boxes of Cocoons were shipped and the required taxes paid on all 10. Additionally, he submits a record from the shipping company, showing that they tried 3 times to deliver the package to Kylie. Can Kylie's attorney now require that Samir produce the remainder of the shipping company's record which shows that their truck only had 6 boxes of Cocoons on it?A) Yes, assuming that Kylie's attorney lays a proper foundation for the introduction of this material.B) Yes, fairness provides that this should be shown at the same time of Samir's introduction of evidence.C) No, unless Kylie's attorney can prove that this is relevant information. Then it can come in.D) No, this is hearsay and inadmissible.
    Kylie's attorney is free to introduce the remaining information contemporaneously with Samir's introduction based on a fairness argument.Note: This is one of the few times that a FAIRNESS-based argument will, in fact, be a correct answer. Remember the particulars of this fact pattern, as generally speaking fairness based answers are incorrect.A is incorrect. It is not necessary for Kylie's attorney to lay any sort of foundation. A fairness argument allows this to just come in.C is incorrect because the standard here is one based on fairness, not relevance.D is incorrect. This is permissible hearsay under the business records exception to the hearsay rule.
  10. A bank is robbed on a Monday morning by a man dressed as a leprechaun. The man is 3 foot 6 inches tall. On Tuesday, the police get a tip from Jon saying that Jake, a person also of short stature, has treated the entire condo complex to drinks at a local bar, bragging how he just struck it rich and flashing a briefcase full of $100 bills. The police go to the condo, knock and Jake answers. The police ask about the rolls of hundreds and Jake states that an old friend of his paid off an old debt regarding a bet they haD) The police leave, only to be called back by the custodian who states that he found a green leprechaun costume and fake beard in the recycle bin. The police return and arrest Jake. After obtaining a search warrant, they search his condo only to find nothing. At trail, the prosecution calls Jon to testify. Jon testifies that about a week prior to the theft his brother was visiting from the Midwest and stated to Jon, "You people here in California sure are a crazy bunch of nuts. Your neighbor across the hall is parading around dressed like a leprechaun." Jake's lawyer objects. The objection should be:A) Overruled, because the statement is a present sense impression.B) Overruled, because the statement is relevant and we have a situation where the declarant is unavailable to testify as having returned to the Midwest.C)Sustained, because this is circumstantial evidence.D) Sustained, because the statement is hearsay not within any exception.
    Here, we have a clear case of hearsay as this is an out-of-court statement offered for its truth, which does not fall within any of the hearsay exceptions.A is incorrect. A present sense impression applies only to situations in which a witness expresses his perception of an event, either while it is happening or immediately after. Here, the time frame is too removeD)B is incorrect. Regardless of the statement's relevance, it is hearsay not within an exception to the hearsay rule.C is incorrect. Generally speaking, circumstantial evidence is admissible unless there is a reason for excluding the evidence. Here, no reason is stated.
  11. ragon is tried for assault after allegedly stabbing Iggy. Dragon represents himself and calls Sooki to the stand, and asks"Sooki, are you familiar with my rep in the town?' Sooki says, "Yep." The prosecution does not object. Dragon than asks, "And what, Sooki, is my rep in town regarding my peaceful behavior?" At which point, the prosecutor objects. Dragon then calls Buffy to the stanD) He seeks to have her testify that the night before the murder Dragon told Buffy that he intended to visit his grandma in Ohio (2000 miles away) and asked whether she would like to join him on the trip. This testimony is:A) Admissible, because it is a declaration of intent to do a future act.B) Admissible, because of the verbal acts exception.C) Inadmissible, because it is not relevant.D) Inadmissible, because it is hearsay that does not fall within any exception.
    Here, we have a hearsay statement that falls within the declaration of intent to do a future act exception. Therefore, it is permissible. In light of this, Answer D is incorrect.B is wrong. A verbal act is an out-of-court statement that when spoken has a legal significance. Here, we are not dealing with a verbal act.C is wrong since the trip could provide an alibi defense and, therefore, is deemed relevant to the case at hanD)Louis is helping Emmett change his oil. As Emmett is jacking up the car, the jack collapses crushing Louis under the engine block. Louis's wife sues Emmett for wrongful death for $10 million dollars. Emmett seeks to testify that he had a conversation with Louis's wife in which she said, "Pay the bills and set me up with a condo in South Beach and I won't sue." Her attorney objects. How should a judge rule on the testimony?A) Inadmissible, because it is hearsay.B) Inadmissible, because it was an offer of compromise.C) Admissible, because it falls under an exception to the hearsay rule since it is a statement against interest.D) Admissible, because it is relevant to show that Louis's wife intended to blackmail Emmett into setting her up with a condo, since she really didn't have an applicable claim to sue on.The evidence here is deemed an admission that is made by, and offered against, a party. This makes Answer A clearly incorrect. Answer C is wrong as a statement against interest is not applicable, since Louis's wife is here to testify. Answer D could possibly be found to be relevant, but it can also be viewed as an offer to compromise. Under a public policy analysis, an offer to compromise or settle a case is generally viewed as beneficial, since it cuts down on nasty court battles, the court's time etc . . .
  12. Kathy is on trial for killing her boyfriend Lee by feeding him into a wood chipper and directing the exhaust of the chipper into a salmon stock pond where the fish could eat Lee's body and grow into strong steelheads. Kathy claims that Lee jumped into the chipper to commit suicide. Kathy's attorney calls Frank to testify that Kathy has a reputation in her community as "a loving person who would not hurt a fly crawling on a coon dog's behinD)" The trial judge should rule the testimony:A) Admissible, since it is not hearsayB) Admissible, as it shows good character in this criminal proceeding.C) Inadmissible, since the prosecution has not yet introduced any information regarding Kathy's reputation.D) Inadmissible; character evidence cannot be used to show that a person acted in accordance with her character.
    Since we are dealing with a criminal case, the accused is free to introduce character evidence to support a finding that she acted in accordance with her character. Therefore, Answers C and D are wrong for misstating the law.A is wrong. The mere fact that evidence is not hearsay cannot be a reason to admit it.
  13. In a personal Injury automobile accident case, the plaintiff's attorney wishes to introduce a sworn deposition taken from Mindy, but Mindy dies a week before the case goes to trial. In Mindy's statement, said that she saw Quentin (the defendant and driver of the car) speed down the road swerving uncontrollably, run the red light and hit the plaintiff's car. (The plaintiff's attorney and the defendant's attorney were both present at the deposition.) The defense attorney objects to the introduction of Mindy's statement into evidence. How should the court rule on the admissibility of the deposition?A) Admissible, the defense attorney had an opportunity to cross-examine during the deposition.B) Admissible, as a dying declaration.C) Inadmissible, because the statement was not made while testifying in a court of law.D) Inadmissible, because the defense attorney has no chance to cross-examine Mindy.
    Mindy's statement is hearsay, as it is a statement other than one made by the declarant while testifying at the trial or hearing, offered in evidence to prove the truth of the matter asserteD) It is admissible, however, under a hearsay exception: namely, the former testimony exception. Under this rule, prior testimony of an unavailable witness (dead Mindy) from a deposition taken in accordance with the law, is admissible in a subsequent trial as long as there is a sufficient similarity of parties and issues discussed, such that the parties had an opportunity to develop a cross-examination during the deposition. In light of this hearsay exception, the present testimony is permissible.B is incorrect. This is not a dying declaration, as Mindy did not believe that her death was imminent, nor did her statements concern the cause or circumstancesof what Mindy believed to be an impending death.C is incorrect. The former testimony rule does not require that a statement be made in court to qualify. A deposition falls within the recognized types of declaration.D is incorrect. The defendant's attorney does not need to have the opportunity to cross-examine Mindy at trial, but only to have had an opportunity to develop Mindy's testimony at the prior proceeding (i.e., the deposition).
  14. Jeanette employs Catty and Kara at her Scrubby Bubbles & Bath store. Kara reports to Jeanette that she saw Catty stealing their "moonlight-scented" body scrub samples and putting them in her underwear. Livid, Jeanette fires Catty and brings a civil action against her for the value of the missing body scrub sample inventory. At the trial of the action, Jeanette calls Kara as the first witness. Kara states, 7 have no recollection regarding seeing Catty stealing from the store, or anything about the missing body scrub samples." Jeanette takes the stand, intending to testify about what Kara had originally told her. Assuming that Catty's attorney takes the appropriate action, Jeanette's testimony would be:A) Admissible, as a statement against interest by Catty.B) Admissible, as proper impeachment of Kara's testimony.C) Inadmissible, as irrelevant.D) Inadmissible hearsay, if offered to prove thefts by Catty.
    Jeanette's testimony would be deemed hearsay if she tries to offer it as prove that Carry stole the body scrub samples and, thus, inadmissible. By definition, hearsay is a statement, other than one made by the declarant while testifying at the trial or hearing, offered in evidence to prove the truth of the matter asserteD) Here, Jeanette's statement is within the definition of hearsay, no exceptions to the hearsay rule apply, and the statement is inadmissible. Note: The statement against interest exception to the hearsay rule applies to statements from a person who is unavailable as a witness and are made against that person's pecuniary, proprietary, or penal interest In the present case, we are not dealing with a statement against interest of the declarant Kara, regardless of the fact that Kara may be deemed unavailable (as she is refusing to testify). The statement of Kara in the present case is against the interest of Catty.Impeachment refers to the undermining of a believed truthfulness of a witness, which shows that a witness has (on another occasion) made statements that are inconsistent with a material part of her present testimony. Here, Kara is merely stating that she does not recall seeing Catty steal or noticing any body-scrub samples missing from the store, and has not made a contradictory statement in a prior proceeding. Generally speaking, Kara's testimony directed toward a lack of memory will generally not be considered to be an inconsistent statement. B is a potentially correct answer (depending on the jurisdiction) if a court believes that the witness is deliberately being evasive in her testimony. Note, however, that D is a better answer that addresses all jurisdictions and is, therefore, the correct answer to the present problem.C is incorrect. Jeanette's testimony is clearly relevant as it tends to make the existence of any consequential facts to the determination of an action more probable than it would be without that evidence. Even though this evidence is relevant, it runs afoul of the hearsay rules and, therefore, cannot be admitteD)A is wrong as a statement against interest, because the statement was not made by Catty, thereby, making the statement against interest exception inapplicable.
  15. Julie is on trial for a criminal offense in which the prosecutor seeks to introduce a tape of Julie's voice to the jury. The prosecutor calls Alec to the stand to authenticate the voice. The only time that Alec has heard Julie's voice was after Julie had been arresteD) Can Alec properly authenticate Julie's voice?A) Yes, since Alec is now familiar with Julie's voice.B) Yes, assuming that Alec is a voice expert.C) No, Alec's testimony would be inadmissible hearsay.D) No, because the only time that Alec heard Julie's voice was after her arrest.
    Alec is free to authenticate Julie's voice if he is now familiar with it, as voice authentication can be by any person, as long as they are familiar with the original voice.B is wrong. An ordinary non-expert can freely authenticate a voice as long as they are initially familiar with it. An expert witness is only required in situations dealing with highly technical or scientific subjects.C is incorrect. By definition, hearsay is a statement, other than one made by the declarant while testifying at the trial or hearing, offered in evidence to prove the truth of a matter asserteD) Here, Alec is not going to be testifying to an out-of-court statement and is solely testifying on whether a recording heard in court is Julie's voice.D is incorrect since Alec's only need is to be familiar with Julie's voice in order to testify. When he acquired that familiarity, is meaningless.
  16. From the following, who will NOT be allowed to verify the handwriting of Carl, who is charged with a forgery?A) An expert witness who looked at both the forgery and Carl's handwriting.B) The jury, when presented with a comparison of writing known to be Carl's and the suspected forgery.C) A police officer who has a copy of Carl's true handwriting.D) A secretary who has worked for Carl for years and seen his signature on numerous letters.
    Here, the police officer is the only person without: 1) personal familiarity with Carl's handwriting, 2) a basis for forming an evaluative opinion of the handwriting derived from specialized knowledge; or 3) a particular fact-finding function, such as that assigned to the jury.A and B are incorrect. An expert witness or the jury can decide on the genuineness of a writing sample when presented with the question of whether or not it is a forgery. Note that the police officer is neither an expert witness nor a trier of fact, making Answer C incorrect.D is incorrect. A lay person with familiarity of Carl's handwriting can be called upon to testify whether the handwriting in question is a forgery.
  17. Ramon seeks to buy a rare first edition book from Nick. The problem is that Ramon only speaks Spanish and Nick only speaks Greek. Ramon asks Jen to speak with Nick on his behalf, since Jen speaks both Spanish and Greek. Jen reaches an agreement with Nick for Ramon to buy the book for $10,000. The agreement is not reduced to writing. Jen tells Ramon all about the agreement, then dies on the spot. Ramon goes to Nick's place of business with a certified check for $10,000 and points to the book, but Nick refuses to turn it over. Ramon sues on breach of contract and seeks to introduce the conversation in which Jen told him, "Nick will sell to you for $10,000." If the jurisdiction has a typical Dead Man Act, what effect will this have upon the admissibility of the conversation?A) Inadmissible because a civil action is involveD)B) Inadmissible because Ramon is an interested party.C) None, as Nick is not a protected party.D) None, because a civil action is involveD)
    Generally speaking, a Dead Man Act provides that an interested party, or their successor in interest, cannot testify to a communication with the deceased person when that testimony is offered against the representative or successor in interest of the deceaseD) Here, Nick is neither a representative, nor a successor in interest of Jen (e.g., an executor, administrator or heir). Therefore, Nick is not a protected party for purposes of the Dead Man Act. Additionally, Ramon's testimony is not being offered against a representative or successor-in-interest for Jen. The Dead Man Act is inapplicable.A is partially correct, but for the wrong reason. While the Dead Man Act applies only to civil cases, the simple fact that there is a civil action does not instantly result in the applicability of a Dead Man Act. As set forth in the correct answer,the inapplicability of the Dead Man Act relates to the absence a protected party.B is wrong. Regardless of the fact that Ramon is a interested party, the Dead Man Act requires both an interested part and a protected party. As set forth in the correct answer, Nick is not a protected party.D is incorrect for stating that the Dead Man Act does not apply to civil cases. In reality, the Dead Man Act ONLY applies to civil cases, not criminal cases.
  18. Summer and Ryan are driving their blue van. Summer is sleeping and Ryan is driving. Ryan gets into an accident and Summer jolts awake, asking "Oh my god! What the heck happened??' Ryan says, 7 ran a red and hit that guy's car" Summer jumps out of the van to inspect the damages. Josh, the driver of the struck car, walks up to Summer and asks, "Are you okay? What happened?' Summer does not realize that Josh is the guy that they just hit and says, "Ryan ran a red and hit this guy's car." At trial against Summer and Ryan, the attorney for the passengers of Josh's car seeks to introduce Josh's testimony which will include Summer's statement. Is this admissible evidence?A) Yes, for the reason of impeaching Summer if she denies having made the statement.B) Yes, because it is an admission by a party-opponent.C) No, Summer lacked foundational knowledge when making the statement.D) No, since Summer had no knowledge that she was talking to an occupant of the struck car when making the statement.
    Here, we have an admission which is non-hearsay under the federal rules. By definition, an admission is a statement that amounts to a prior acknowledgment by a party to a relevant action or fact in issue. Such a statement would ordinarily be deemed hearsay, but a court will permit its introduction on the theory that if the party said or did something that now turns out to be inconsistent with her contentions at trial, she should be stopped from preventing the earlier statement's admission into evidence.A is incorrect. It is impossible to impeach a witness before they have testifieD) Here, Summer and Ryan have yet to present their case, and Summer has not yet testified, making her impeachment impossible.C is incorrect. Lack of personal knowledge does not absolutely result in the exclusion of a party's admissions.Note: Summer may try to discredit the statement during Ryan's case in chief, on the basis that she lacked any knowledge when the statement was made. The introduction of the statement cannot, however, be absolutely preventeD)D is incorrect. Whether the party knew they were speaking to a potential trial opponent at the time of the admission, has no bearing on the statement's admissibility by a party-opponent.
  19. Bear is a mechanic at a dealership who often test drives the cars that he fixes to be sure that everything works. One day, after fixing a LeDorien, Bear gets into a huge accident while speeding and showing off for some girls. The accident is with Boyd, who failed to stop at a red light. Boyd is thrown out of his car and lands in a pen of pigs. His car is totaleD) Bear realizes that Boyd is really hurt and runs to the pig pen to help him. Bear apologizes profusely saying, 7 did a rush job on this LeDorian and didn't fix the brakes as well as I usually do. I am SO sorry." Boyd brings an action for damages against the dealership where Bear works. At trial, Boyd's attorney calls a bystander to testify about the statement made by Bear. The dealership's attorney objects. How should the court rule?A) Sustained, because Bear's statement is inadmissible against the dealership.B) Sustained, since Boyd failed to stop at the red light.C) Overruled, because it is a declaration against interest.D) Overruled, because it is an admission of a party-opponent.
    Here, we have an admission of a party-opponent which is admissible under the federal rules. By definition, an admission is a statement made or an act done that amounts to a prior acknowledgment by one of the parties to a relevant action or fact. Note: An admission statement need not be against an interest at the time that it is made. In the present situation, the statement of a bystander is being used to prove the truth of the matter asserted (namely, that Bear failed to properly fix the brakes) and would ordinarily be inadmissible. However, this statement was made by Bear himself while working as an agent for the dealership in regards to a matter within the scope of Bear's duties. Therefore, the introduction of the statement, as an admission by a party-opponent relating to the negligence in the repair of the brakes, is admissible.A is wrong. Bear's statement is admissible against the dealership as a vicarious admission.B is wrong because it is irrelevant to the issue of whether the bystander's testimony is admissible. Boyd's failure to stop has no relation to Bear's admission of the bystander's statement.C is wrong. The statement against interest exception to the hearsay rule only applies when the declarant is not available as a witness. Bear is available as a witness. Furthermore, a statement is admissible as an admission by a party-opponent even if not against interest when made. This means that Bear's against interest admission is admissible even if it does not negatively effect the dealership at the time that it was made.
  20. Kirsten is run over by a speeding car driven by Seth. Officer Copper arrives on the scene to find Kirsten drifting in and out of consciousness. Kirsten mutters, "Why didn't he stop...he ran the light, and now I'm dying....Why didn't he stop...he ran the light." Officer Copper makes a note of her comments. The officer tracks down Seth and Kirsten surprisingly lives to walk again, only she is harbored by a huge mental disability. Kirsten sues Seth, but before trial she dies of a drug overdose unrelated to the accident. This jurisdiction allows for survival of personal injury actions. Thus, Kirsten's estate steps in the place for Kirsten as plaintiff. Her estate seeks to have Officer Copper testify regarding Kirsten's statements at the time of the accident. Will a court permit this?A) Inadmissible, because Kirsten's death was not caused by the accidentB) Inadmissible, because this is a civil case and not a criminal matter.C) Admissible, because the statements were Kirsten's present sense impressions.D) Admissible, because the statements were made at a time when Kirsten thought death was imminent.
    Testimony of Officer Copper is admissible. Since Kirsten's statements were made while she thought death was imminent, they qualify under the dying declaration exception to the hearsay rule. Statements regarding the cause of death (or potential cause of death) are admissible, under the dying declaration exception to the hearsay rule, when faced with a statement (by a now unavailable declarant) made while the declarant believed she was in danger of imminent death;Note: The actual death of the declarant is not absolutely necessary, only fear of imminent death at the time the declarant made the statement in question. Under the dying declaration rule, the declarant does not need to be dead at trial, just unavailable for questioning. In light of this, Answer A is clearly incorrect.B is incorrect for stating the traditional rule of dying declaration, wherein a dying declaration was only applicable in homicide cases. The federal rules, unlike the traditional rule, allows declaration in both civil cases and homicide prosecutions.C is incorrect. Kirsten's statement is not awarded protection under the present sense impression exception to the hearsay rule, as a present sense impression relates to a comment made while watching a non-serious event that is neither shocking, nor exciting. Here, getting run over by a car is clearly shocking and exciting.
  21. Isamu and Kaitlin crash into each other in their SUVs causing both to flip over. Stern happens upon the accident. As he rushes to the scene, he sees all of the SUVs* wheels spinning away and distinctly notices that Isamu's wheels are spinning much faster than Kaitlin's wheels. Kaitlin's attorney calls Stern to the stanD) Stern is a 20 year veteran mechanic for a racing team pit crew. He testifies that he happened across the accident and, as he rushed to the scene to help, saw all the SUVs flipped over with their wheels spinning. Kaitlin's attorney then asks Stern to estimate the speed of the spinning wheels. Isamu's attorney objects. Should the testimony be admitted?A) Yes, as Stern's personal opinion.B) Yes, as a matter based upon personal observation.C) No, unless Stern is a qualified expert in accidents such as this.D) No, unless there is another witness to corroborate that Stern was actually at the scene.
    The testimony is only admissible if Stern can be found to be an expert in accident reconstruction such as this, regardless of his racing mechanic backgrounD) When faced with a situation where technical or other specialized knowledge is required to render an opinion, a court will only allow expert testimony and will prevent testimony from a layperson. Experts require a showing of special knowledge or expertise in the facts at hanD) Since Stern is going to testify about the speed of the vehicle based on the spin of the wheels, he is acting in the capacity of an expert. To testify as an expert, Stern musty possess special knowledge regarding accident reconstruction. Being a racing mechanic is probably not enough and, unless Stern is found to be an expert, his testimony is not acceptable.A is incorrect. If Stern is not found to be an expert in accident reconstruction, his opinion is just an opinion and not the testimony of an expert. Opinion is not admissible in the present case.B is incorrect. Again, unless Stern is an expert, his opinion based on the personal observation of the spinning wheels will not be permitted to come into the case.D is incorrect because the presence of a witness at the scene of events to which his testimony relates need not be corroborated by another witness.
  22. Amita is walking down the street when she sees two cars (driven by Keystone and Davidson) crash into each other. Keystone sues Davidson. Keystone's attorney calls Amita as a witness and asks her to describe the scene of the accident, the weather, and the time of day. Davidson's attorney then asks the same of Amita, and follows up by asking her if either car burst into flames. Amita says, "Yep, both were a fireball, just like in that movie about the convoy trucks," and then blurts out, "They had to be going at least 60mph eachr Davidson's attorney moves to strike that last statement by Amita concerning speeD) How should the court rule?A) Strike it, as unresponsive to any question askeD)B) Strike it, because Amita is just a pedestrian with no way of determining car speeds.C) Overruled, because Davidson's attorney has opened the door to anything Amita might say.D) Overruled, since Amita's statement accuses both cars of going 60 mph and is not prejudicial to either party.
    Amita's answer is unresponsive to the question asked and the court should strike it. When dealing with an unresponsive answer, the examining counsel can strike, but the opposing counsel CANNOT. Here Davidson's attorney asked a pointed question requiring a detailed answer. Amita, in return, answered the question but added additional info not requested (i.e., was unresponsive). Since Davidson's attorney asked the question, she is free to motion to strike the unresponsive portion of Amita's response.B is wrong for two reasons. First, Amita's comment regarding the speed of the cars should be struck as unresponsive, even if Amita was a skilled expert on vehicle speeD) Furthermore, Amita's response as a pedestrian could be permissible to show the speed; a lay person is free to provide an estimate as to vehicle speed so long as one can establish that the lay person has some understanding and experience regarding relative speeds of vehicles. If we can establish that Amita has this experience, testimony directed to speed of the vehicle would be permissible.C is incorrect for misstating the "opening the door" concept. "Opening the door" relates to one who introduces evidence on a subject. A defendant or plaintiff cannot complain if their adversary goes on to offer more evidence on the same subject, or questions the witness on the same subject (absent an objection related to something other than relevance), if that defendant or plaintiff originally introduced the subject during their own line of questions. The motion to strike is based solely on the unresponsive answer; not based on Keystone's attorney opening the door.D is incorrect as a prejudicial answer that was non-responsive is NEVER the basis for a motion to strike. While most examining attorneys will strike an answer as unresponsive because it is prejudicial to their side, the move to strike an unresponsive answer is based solely on the unresponsive nature of the answer and not on the prejudicial factors associated with it.
  23. A state court is least likely to take judicial notice of which of the following?A) O Positive is the most common blood type in the US population.B) Highland Avenue, on which the home of Hudson is located, runs North/South.C) The sun rose at 5:41 AMPST on March 29, 2005.D) In Polish law, there is no private action for issues revolving around un-pasteurized vodkA)
    The state court will unlikely take judicial notice of a Polish law, as taking judicial notice of a foreign law is unlikely.A and B are deemed notorious facts (facts known by the community) and are permissible.C is a manifest fact that can easily be verified by using easily obtained sources that are readily accessible and verifieD) Both manifest and notorious facts are appropriate for judicial notice, and under the federal rules.Note: If a party requests notice of these facts, a court MUST take notice.
  24. uppose that the Somerville Daily Mail prints the following on its front page, "The best description of Bobby, Jr. is a smelly alcoholic and everyone knows it" Bobby, Jr. promptly sues for defamation. During the case, Bobby's attorney seeks to put Joe on the stand, to testify that Bobby once saved a fellow passenger from falling out of a tourist duck boat and into the Charles River. Assume that the newspaper's lawyer objects to this testimony. The court should rule:A) Admissible, plaintiff is free to introduce evidence of his good character.B) Admissible, because character has been brought into question by the paper.C) Inadmissible, because Joe's testimony is not probative of any material issue.D) Inadmissible, because a party cannot offer specific instances of conduct to prove character.
    Joe's testimony will not be admitted as it offers no probative value related to a material issue in the case (i.e., whether Bobby, Jr. is actually a "smelly alcoholic"). At best, Joe's testimony offers up the fact that Bobby is heroic and brave, but does not relate to the "smelly" or "alcoholic" claim made by the Somerville Daily Mail.A is incorrect. Character evidence can only be admitted when a person's character is directly an issue, but the evidence must be relevant to the particular character,traits in issue (i.e., smelliness and alcoholism).B is incorrect for the same reason as Answer A) In order for evidence to be admissible, the evidence must be relevant.D is incorrect because a court will allow proof of specific instances of a person's conduct in situations where their character is directly in issue.
  25. Marshal sues Micah on the basis that Hudson, Micah's dog, has bitten Marshal for no reason. At trial, Marshal's attorney calls Marshal to the stand and after asking questions about date of birth and home address asks, 'Were you bitten by a brown dog wearing a Louie Mutton collar?' Marshal answers yes and is dismisseD) Marshal's attorney then calls Micah as an adverse witness. Marshal's attorney asks some simple questions like name and address, and then asks, "Does your dog have brown fur and wear a Louie Mutton collar?' Micah answers yes, and Marshal's attorney has ho more questions. Micah's attorney questions Micah while on the stand and asks, "Has Hudson shown anything but a gentle loving demeanor with all animals great and small?' Marshal's attorney objects. How should the court rule on this objection?A) Sustained, because the defense is attempting to introduce character evidence when character has not been called into questionB) Sustained, because the defense's cross examination goes beyond the scope of a property cross examination.C) Overruled, because the plaintiff brought up the dog in direct examination.D) Overruled, because the testimony sought is relevant and otherwise admissible.
    The testimony sought is both relevant and admissible as admissible evidence is such that it tends to make the existence of a material fact more probable or less probable than it would be without that evidence. This question will make it less or more probably that Hudson was the dog that bit Marshal, making this evidence relevant. Relevant evidence is admissible unless there is some rule against its admission (e.g., hearsay). Here, there is no relevant rule preventing it.A is wrong because character evidence relates to human traits, not to character traits of animals.B is incorrect since the direct examination was related to the dog that bit Marshal, and this question is clearly related to that same issue. A party can cross-examine on any matter that came out during the direct examination, or any inference drawn from a matter that came out on direct examination. The inference, here, was that Hudson was the dog who bit Marshal. On cross examination, Micah's attorney is simply trying to show that it was not Hudson, as Hudson has a gentle demeanor.C is wrong since the simple mention of the dog on direct does not mean that anything dog related can be brought up on cross examination. On cross examination, the questions must be relevant and related to the matter brought out during direct examination.
  26. Barnard is hit by a car driven by Ralph. Barnard is rushed to the hospital where he is treated for some bruises and released within an hour by Dr. Vigus. Barnard then visits his family doctor, Dr. Frank. Barnard sues Ralph and at trial testifies about his sever back pains and whiplash. Ralph's attorney subpoenas Dr. Vigus, questions Dr. Vigus about her hospital affiliation and then asks her to describe Barnard's condition when he was brought in to the emergency room. Dr. Vigus is about to answer when Barnard's attorney objects, stating that Barnard wishes to invoke the doctor/patient privilege. How should the court rule on the objection? A) Sustained, because the patient has the right to invoke the privilege.B) Sustained, because Dr.Vigus's testimony has no bearing on Barnard's present condition.C) Overruled, since Barnard is suing for personal injuries.D) Overruled, because Dr.Vigus was not Barnard's doctor prior to the accident.
    When suing for personal injuries, a patient cannot invoke the doctor/patient privilege as the patient has put his personal injury at issue in the case.A is wrong, even though it is usually the patient who decides when to invoke the doctor/patient privilege. As mentioned, the privilege is not applicable once the patient, has put his personal injury at issue through suit and cannot, therefore, prevent the introduction of personal injury information form Dr. Vigus.B is wrong. Dr. Vigus's testimony is relevant since the evidence she has to offer may determine whether an action of fact in question is more probable than it would be without her evidence. In other words, Dr. Vigus's testimony would tend to help ascertain if Barnard did, in fact, receive a back injury from the accident.D is incorrect. The doctor/patient privilege does not only stem from a patient and existing physician relationship. It stems from any patient/doctor examination, regardless of whether the physician was the patient's doctor before the exam.
  27. The police arrest Jack for the murder of Dawson. The police go to Jack's house, where they find Mrs. Ripper sitting watching The Dr. Gil Show. Mrs. Ripper is Jack's wife. The police ask Mrs. Ripper to give them the jeans that Jack wore to work on July 7th, the evening that Dawson was killeD) Not saying a word (as Mrs. Ripper is totally engrossed in the show's topic of spouses who lead double-lives) she leans over and picks up a bloodstained pair of boot-cut jeans and tosses them to the police. They test the jeans and find that they are covered with Dawson's blooD) At trial, the prosecutor seeks to introduce this evidence into trial. If the defense objects, the court should rule that the jeans are:A) Admissible, as relevant evidence linking Jack to the crime.B) Inadmissible, because of the marital privilege.C) Inadmissible, as hearsay not within any exception.D) Inadmissible, because of the privilege against self-incrimination.
    The jeans are admissible as relevant, as they were offered in an unobjectionable manner that is not in violation of one of the rules of evidence. As the jeans make it more probable that Jack committed the murder; they are relevant and admissible.B is incorrect since spousal immunity or privilege only applies to prevent the forcing of a spouse to testify against the other in a criminal proceeding. Based on the facts, that is clearly not the case. The marital communications privilege does protect statements made between married partners in confidence, but here we have no communications between married partners. Additionally note that these privileges are testimonial privileges, which probably would not prevent the casual turning over of evidence from Mrs. Ripper when asked by the police for Jack's jeans.C is incorrect. The jeans cannot be viewed as a statement under the hearsay rules, and the hearsay rules only apply to statements offered for the truth that are made outside of court.D is wrong for two reasons. First, this privilege against self-incrimination only applies to testimony, not the handing over of evidence. Second, the pair of jeans turned over by Mrs. Ripper does not incriminate her, but rather incriminates Jack.
  28. Negresco sues Jeanne in a contract dispute over some gold lame toilet seats. Negresco's attorney calls Henry to testify about his personal knowledge of his friend's agreement. He then seeks to have Cruz testify about how Henry, a priest, is honest through and through. Jeanne's attorney objects and the court sustains the objection. Negresco's attorney cannot bring Cruz to testify abut Henry because:A) Henry's credibility was never in question.B) Cruz's testimony would be inadmissible under the hearsay rule.C) Cruz cannot testify regarding an opinion of honesty.D) Evidence relating to a person's reputation is generally not admissible in a civil case.
    Here, Henry's credibility was never an issue, so there is no reason to allow Cruz to testify regarding Henry's credibility. Bolstering a person's credibility is only permissible when the credibility of a person has been questioned via an impeachment of that person.B is incorrect. There appears to be no hearsay issue based on the given facts. By definition, hearsay is an out-of-court statement offered in evidence to prove the truth of a matter asserteD) Here, Cruz would not be testifying to an out-of-court statement, but rather to Cruz's known opinion of Henry.C is wrong. A witness's credibility can be supported (or attacked) based on opinion evidence regarding the witness's honesty. If Henry's honesty was under attack, Cruz's opinion could be introduced into evidence.D is incorrect since a witness's reputation for truthfulness can, in fact, be introduced into a civil case for impeachment purposes. It is character that generally cannot be introduced into a civil case, unless character itself is in issue in the case.
  29. Suppose that Flighty Airlines flight #1 crashes into a telephone pole while accidentally flying too low over LouisianA) Charles, a passenger on the flight, is killed when the plane crashes. Buddy is Charles's executor who sues in state against Flighty Airlines as well as Bowstring, the manufacturer of the plane. The FAA investigation regarding the crash has determined that it was due to pilot error, since the pilot was accidentally working in metric units and not imperial units. The pilot was also killed in the crash. Buddy seeks to introduce the FAA report into evidence. Should the judge allow it?A) Yes, because it is a public recorD)B) Yes, but only for impeachment purposes.C) No, it is hearsay and the dead pilot is unavailable to testify.D) No, because of the Best Evidence Rule.
    Here, we do have a potential hearsay problem, but the FAA report falls clearly within the public records exception to the hearsay rule. Hearsay is an out-of-court statement offered in evidence to prove the truth of the matter asserteD) The public records exception to the hearsay rule states that records, or reports from a public agency, are admissible if they set forth: 1) the activities of the office or agency; 2) matters observed pursuant to a duty imposed by law or 3) factual findings resulting from an investigation made pursuant to authority granted by law in civil actions, and against the government in criminal cases. Here, the FAA report can be viewed as hearsay as it is an out-of-court statement offered to prove the truth of the matter asserteD) However, we are dealing with the result of an investigation made pursuant to authority granted by law, contained within a public agency compiled report, allowing applicability of the public agency exception.B is incorrect. The FAA report can come in as either substantive evidence or evidence to impeach.C is incorrect. The report is admissible as a hearsay exception. The fact that the pilot cannot testify is meaningless, as it has no bearing on the fact that the report can be admitteD)D is wrong since the Best Evidence Rule does not apply to the present facts. Best evidence applies to a situation where one is proving the terms of a writing where the terms are in question, therefore, the writing itself must be supplieD) Best evidence does not apply to a situation, such as this, where the fact to be proved (material fact in the writing) exists independent of the writing. The intended fact to be proved (pilot negligence) does not depend on the FAA report for substantive support.
  30. Richie is driving north on Easy Street, when the steering wheel on his car randomly breaks off and falls into Richie's lap. He looses control of the car. The car crosses the double yellow line and runs into a car driven by Cadbury (who was heading south on Easy Street). Reggie, a recently laid off news reporter, stumbles across the accident and begins snapping photos of the accident with her cell phone camerA) (Up until now, Reggie just used this camera to snap pictures of good looking guys to email to her younger unmarried sister.) Cadbury sues Richie for damages, intending to introduce Reggie's photos. Are the photos admissible?A) Yes, but only if Reggie is also a police office who took the photos as part of an official report.B) Yes, but only if Reggie can testify at trialC) No, unless a proper foundation is laiD)D) No, because of the Best Evidence Rule.
    The photograph is not admissible unless a proper foundation is laid, such as identification by a witness that that the photos represent the accident scene. For example, a witness familiar with Easy Street would need to be called to verify the scene and lay the proper foundation.A is incorrect. Photos taken by one other than a police officer for an official report are, in fact, admissible assuming the proper foundation is laiD)B is incorrect because the photographer is not needed to authenticate the photos. Any witness familiar with the scene can authenticate.D is incorrect. The Best Evidence Rule does not apply here, as the Best Evidence Rule states that in proving the terms of a writing where the terms are material, the original writing must be produceD) The writing, here, can be viewed as the photos. Secondary evidence, under the Best Evidence Rule, is only permitted when it is shown that the original is unavailable. Here, a copy of the photo is not being offered, making the Best Evidence Rule inapplicable.
  31. Suppose that Alissa is charged with robbing the LiquorMeUp liquor store on December 4th in Kansas. Alissa bases her defense on the fact that she was not in Kansas on December 4th and seeks to introduce an email written to her boyfriend that states, "I'll see you on my birthday at our favorite restaurant in Boston" It is ascertained that Alissa's birthday is December 4th. The email is:A) Admissible, as evidence of Alissa's intent to be in Massachusetts on December 4th.B) Admissible, as a present sense impression.C) Inadmissible, because the statement in the email is irrelevant.D) Inadmissible, because it is hearsay not within any recognized exception to the hearsay rule.
    The email is admissible as evidence of Alissa's intent to go to Massachusetts on December 4th, but nothing more. By definition, hearsay is an out-of-court statement offered to prove the truth of the matter asserteD) Alissa's email is an out-of-court statement, and it is being offered to prove its truth, making the email hearsay. However, an exception to the hearsay rule, namely a state of mind declaration, is applicable. Under the state of mind declaration, the email can be admitted as a showing that the declarant may have carried out the stated intent (i.e., permissible circumstantial evidence).B is wrong for applying the incorrect hearsay exception. A present sense impression is a comment made while an ordinary event is occurring, wherein the event occurring is neither exciting nor dramatic in any way. This is not the case in the present fact situation, making this exception to the hearsay rule inapplicable.C is wrong. The email is relevant, as any evidence that tends to make the existence of a material fact more probable than it would be without the evidence is deeded relevant by a court of law. Here, Alissa's location on December 4th is certainly relevant to the pending case.D is wrong in light of the correct answer, as the letter falls within the state of mind exception for the hearsay rule of law.
  32. Suppose that the state arson law closely mimics the common law rule, with the exception that the "dwelling house of another" portion of the common law rule has been removeD) Byrne is charged with arson for burning down his own house. Byrne's attorney calls Franz as a witness at trial. Franz testifies that he was at Byrne's house when the fire started and that the conflagration started when embers from the fireplace popped out of the firebox and lit the polyester shag carpet ablaze, eventually burning down the house. Franz further testifies that the fire was "a total freak accident" and that Byrne and Franz were doing nothing wrong. The prosecution then asks Franz, "Mr. Franz, aren't you being charged with first degree arson in relation to this same house in another proceeding?1 The defense objects vehemently. Should the court allow the prosecutor's question?A) Yes, assuming Byrne's attorney has introduced something relating to the good character of Franze.B) Yes, the question is applicable to a showing of Franz's bias or interest in the witness.C) No, because Franz has not been convicted of the crime.D) No, because the question violates Franz's Fifth Amendment right to be protected from self incrimination.
    The court should allow the question, as it is applicable to a showing of bias or interest on the part of the witness. Such a showing tends to illustrate that Franz has a interest in lying about the case due to his bias. One can show a bias by cross-examination, extrinsic evidence or both. Since Franz is being tried for the same crime, it is correct to admit this question to show Franz's interest in the pending case.A is wrong. Any witness can be impeached, and evidence introducing facts relating to truthfulness or veracity of the witness is always allowed to come in.C is wrong. A showing that one has convicted a crime is not a prerequisite for a showing of bias or interest evidence. Showing that a person has been convicted of a crime or felony, is an entirely separate way to impeach the witness (or show bias).D is wrong since answering the question does not incriminate Franz, but only shows that he is being charged elsewhere. If somehow answering the question could tie Franz to the crime, he has the right to refuse to answer under his Fifth Amendment rights. The question itself,however, is permissible.
  33. Dorothea is a physician's assistant working at the Kirkbride Mental Asylum. There is a tort case charging the asylum and Dorothea is called to the stanD) At issue is whether Gait, the plaintiff, ever lost consciousness during his shock therapy regiment. Gait's attorney seeks to introduce testimony from Dorothea (who was working at the voltage dial during the time in question) that she believes Gait did, in fact, lose consciousness. Would such testimony be admissible over the objection of the Kirkbride Mental Asylum's attorney?A) Yes, because it is proper opinion testimony by a lay witness.B) No, because Dorothea is not an expert.C) No, because it hinders the jury's determination in the case.D) No, because it is not the best evidence.
    Testimony by Dorothea is proper opinion testimony by a lay witness and is admissible. Lay opinion testimony is admissible when: 1) it is rationally based on the perception of the witness; 2) it is helpful to a clear understanding of her testimony or to the determination of a fact in issue; and 3) it is not based on scientific, technical, or other specialized knowledge. Here, Dorothea is simply being asked to describe Gait's appearance and consciousness. Such an opinion is not specialized or technical and any ordinary person can answer. Therefore, lay testimony such as this is permissible.B is wrong. Expert testimony is not applicable, here, as Dorothea is simply testifying to the general appearance of Gait, which can be done by an non-expert witness. Expert witness testimony is often required when dealing with a highly technical fact pattern, but such a situation is not applicable here. All we are asking is if Gait was conscious or not, which any lay person can answer.C is wrong. Dorothea's testimony at minimum helps the jury, rather than hinders the jury, in the determination of a disputed fact.D is wrong. The Best Evidence Rule applies to a situation where the terms of a writing, which is material, is in dispute. In such a situation, the Best Evidence Rule requires that the original writing be produceD) Here, we have no writing, just testimony of what Dorothea saw.
  34. Daft sues Bugzy in relation to a neck injury that occurred when Daft and Bugzy ran into each other on their scooters. Daft asserts that Bugzy ran a stop sign and hit him. Daft seeks to call Elmer (who was near the stop sign at the time) to the stanD) Elmer is ready to testify that Bugzy offered to pay him $1,000 to keep his mouth shut and testify on Bugzy's behalf instead of Daft's behalf. Such testimony should be:A) Admitted, as substantive evidence regarding the weakness of Bugzy's case against Daft.B) Admitted, to impeach Bugzy for specific bad conduct.C) Excluded, since it is irrelevant.D) ExcludeD)Even though it is relevant, it is unjustly prejudicial.
    Bribery evidence such as this is clearly admissible against Bugzy. By definition, an admission is a statement made that amounts to a prior acknowledgment by one of the parties of one of the relevant facts. Party admissions are never deemed hearsay. Conduct such as this bribery, is traditionally viewed as an expression of guilt by a party and in the present case Bugzy's expression of guilt should be permitted as an admission of a party-opponent.B is wrong in light of the correct answer as this is an admissible admission of a party-opponent and not admissible as an impeachment. Additionally, a specific act of misconduct used to impeach can only be used on cross-examination. Here, we have no facts indicating that Bugzy has ever been on the stand to testify.C is wrong. The bribery offer is certainly relevant, as it tends to make the existence of a fact of consequence to the action more probable than it would be without that evidence. As set forth above, bribery tends to indicate guilt and is clearly relevant to the present case.D is wrong. The simple fact that the evidence is misleading or prejudicial is not reason enough to prevent its admission. Admissibility of potentially prejudicial evidence is weighed by the judge who has discretion over whether to allow the evidence to be introduced into testimony. The judge weighs the evidence's prejudicial nature vs. it's benefit in relation to the case when making his decision.
  35. Foxy runs into Randy with her SUV, leaving Randy with several injuries. The police arrive and require Foxy to take a breathalyzer test. She is found to be drunk and is citeD) She goes to court and is convicteD) Foxy gets the maximum penalty: 90 days in jail and her license revokeD) Randy sues seeking compensation for his neck injuries. At trial, Randy's attorney offers a copy of the judgment against Roxy, properly authenticated, to be introduceD) The evidence is:A) Admissible as a public recorD)B) Admissible as a final judgment offered to prove a fact that is essential to the present controversy.C) Inadmissible, because the maximum penalty for the crime was 90 days.D) Inadmissible, because it is not the best evidence.
    The federal rules state that judgments of felony convictions are admissible, but define felony convictions as crimes that are punishable by death or imprisonment in excess of one year. We are, therefore, not dealing with a felony and Roxy's conviction is inadmissible hearsay as proof of the fact asserteD)A is incorrect. By definition, a public record is a record, report, statement, or data compilation of a public office or agency setting forth: 1) the activities of the office or agency; 2) matters observed pursuant to a duty imposed by law; or 3) factual findings resulting from an investigation made pursuant to authority granted by law. Here, we are not dealing with such a record, making the public record based answer incorrect.B is incorrect. A final judgment that is offered to prove the facts of the present controversy could be admitted, as long as it is a felony under the federal rules (over 1 year of imprisonment or death).D is incorrect since a properly authenticated copy of the court report is the best evidence. The Best Evidence Rule states that, in proving the terms of a writing where the terms are material, the original writing must be produceD) Duplicates are admissible under the Best Evidence Rule unless: 1) the authenticity of the original is challenged; or 2) circumstances exist that would render it unfair to admit the duplicate in place of the original. In light of this, a properly authenticated photocopy of the prior judgment would be permissible and not violative of the Best Evidence Rule.
  36. Yuliana and Jay help Candace unload boxes of her books onto a loading dock in front of her new apartment. Candace loves books and is using a forklift to carry r^r heavy collection. As she is driving the forklift, she collides with Kendra in her car. As they collide, the load of books tumbles to the ground and falls on Eve, who was walking in the roaD) Eve is hurt. A shopkeeper across the street sees all this happen and calls the police who arrive to take statements. The shopkeeper has to leave shortly after the police arrive at the scene to pick up his kids from school. Subsequently, he does not make a statement to the police until the day after the accident. The shopkeeper did, however, joy down everything he saw, including license plate numbers, just after he called the police. Kendra sues Candace for negligence, claiming that Candace should not have been driving a forklift in the roaD) Kendra also asserts that she was driving in observance of all the traffic laws. Candace claims that Kendra actually swerved into her and the forklift, because Kendra was distracted on her cell phone. All of the witnesses are available to testify at trial. At trial, the shopkeeper has a tough time recalling all of the facts and Candace's attorney seeks to have him refer to the notes he took at the time of the accident. How should the court rule?A) Permissible, it is a present recollection refresheD)B) Permissible, because it is a past recollection recordeD)C) Impermissible, a business owner is obligated to report all accidents to the local police department, as well as the landlord of his establishment, and the record is, therefore, privilegeD)D) Impermissible, as we have no evidence to suggest that the shopkeeper used the notes when giving his statement to the police the following day.
    The rule of present recollection refreshed allows a witness to refer to any writing which serves to refresh his memory of an event.B is wrong. Past recollection recorded requires a proper foundation for introduction and only applies to the admissibility of the contents of a writing. It is inapplicable here.C is an obviously wrong answer. What exactly is this supposed to mean given the present facts? Every once and a while, the test-makers will include a totally bogus choice like this one. Be thankful and instantly rule it out.D is wrong as the shopkeeper is testifying to the recollection of the accident, not to the contents of the writing he put together at the time of the accident.
  37. Alexander is being charged with the rape of Lock. Alexander testifies that 1 week before the rape he was out at sea, working on a fishing boat off the coast of Maine, making it impossible for him to have committed the crime. Lock's attorney introduces the testimony of Arabella, a bartender who testifies that Alexander tried to pick her up by offering Arabella a $100 dollar bill the evening before Lock's rape. The bar is located in the same town that Lock lives in. Should Arabella's testimony be admitted?A) No, because the character of Alexander is not in issue.B) No, because the evidence offered pertains to a prior uncharged offense.C) Yes, because it is offered to show that Alexander was, in fact, in the town near Lock's apartment around the time of the rape.D) Yes, because it shows that Alexander was after sex around the time of the rape.
    Arabella's testimony is probative evidence which relates to the issues of the case and, therefore, should be admitteD) Since the question of Alexander's location is an issue, and Arabella's testimony makes his location clear, it should clearly be admitted as requisite testimony in the case as it is relevant.A is incorrect. Although true that Alexander has not put his character in issue, the testimony of Arabella is not being offered to prove Alexander's bad character, but rather to show his whereabouts near the location of the crime. Therefore, Arabella's testimony is, in fact, admissible.B is wrong. First: prior crimes can be used to show more than bad character and a criminal character. They can be used to show intent, opportunity, or a plan. In light of this, Alexander's offer of money for sex is not instantly prevented from introduction. Second: Arabella's testimony is not being offered to show bad character, but rather to place Alexander in proximate location to the scene of the crime, rather than far away in the North Atlantic as he claimeD) In light of this, the testimony is permissible.D is incorrect. Evidence illustrating a defendant's criminal disposition and that alone, is generally not admissible. However, as mentioned, testimony offering to show more than this could, in fact, come in.
  38. Barney is charged with being one of 5 masked men who robbed the Caton Bank of Wisconsin at gunpoint at 1pm on December 28th. Barney's attorney calls Fred to his defense. Fred states that he picked up Barney and his wife at 11:00 am and went fishing with them until 10:00 pm that night, the night of December 28th. On cross-examination, the prosecution asks Fred, "And what is your relationship to Barney's wife?" The defense counsel objects. How should the court rule?A) Overruled, because the question attacks the witness's truth and veracity.B) Overruled, because the question is directed at discovering possible bias in the witness.C) Sustained, because the question seeks to elicit irrelevant information.D) Sustained, because the answer to the question could create prejudice that would outweigh its probative value.
    The prosecutor's question is permissibie to illustrate a bias, and that Fred has a motive to lie. Here, the prosecutor is trying to impeach Fred; namely, to cast an adverse reflection on Fred's veracity. Bias (or an inference of it) can be shown by examining a family relationship. Here, the prosecutor may potentially be trying to show that due to a familial relationship between Fred and Barney's wife (i.e., brother and sister?); Fred might be inclined to lie for Barney.A correctly states that Fred's truth and veracity is attacked, but not all methods of attacking a witness's truth and veracity are admissible. In light of this, Answer B is a better answer since it specifically states the exact method the prosecutor is using to impeach the witness.C is incorrect because evidence relating to a witness's credibility is in fact relevant.D incorrectly states that an answer by Fred would result in a prejudice that would outweigh its probative value. The federal rules state that evidence may be excluded if its probative value is substantially outweighed by the danger of unfair prejudice, confusion of the issues, or misleading the jury. Note that only the unfair prejudice is addresseD) Here, the prosecutor's question will readily result in an answer as to whether Fred has a motive to lie (e.g., a family relationship) and, therefore, is not unfairly prejudicial.
  39. Astro is charged with burning down Elroy's house. Elroy is Astro's brother. During an arson investigation, it is determined that diesel fuel was used as an igniter in the fire. The prosecution calls Judy, the owner of a 24hr convenience store that is near Elroy's house. Judy will testify that on the night of the fire Astro bought 10 slim jerkys and a diet colA) She will also testify that Astro smelled like diesel fuel. Should the court admit this testimony over the defense's objection?A) No, it is inadmissible as the opinion of a non-expert witness.B) No, it is inadmissible because the best evidence is the result of the chemical tests.C) Yes, it is an admissible lay opinion testimony.D) Yes, it is admissible expert testimony because anyone who drives a car is an expert as to the smell of fuel.
    Judy should be allowed to testify as the evidence is probative of a material issue in the case. Note: Evidence is material if it relates to an issue in the case, and it is probative if it tends to prove the fact for which it is offereD) Evidence is competent if it does not violate a specific exclusionary rule. We have an issue of whether Astro started the fire. If we assume that diesel was used to start the fire and Astro was found to smell like diesel, there is a likelihood that he was involved in the fire, so the proffered evidence is material and relevant. Addressing the competency question, opinion testimony is admissible when: 1) it is rationally based on the perception of the witness, and 2) it is helpful to a clear understanding of her testimony or to the determination of a fact in issue. Courts have traditionally allowed something as simple as smell testimony to be allowable testimony by a lay witness. In light of this, Answer A is incorrect.B is incorrect. The Best Evidence Rule requires that the original document be produced, only when the terms of the document are material and sought to be proveD) Here, the fact to be proved is that diesel was used, which stands independent of any document. Therefore, the Best Evidence Rule is inapplicable.D is incorrect since expert testimony requires special knowledge or skill level. Here, we have a simple question of smell, and anyone who has ever pumped gas or gotten gas knows the smell of diesel fuel. There is nothing special about this knowledge.
  40. Suppose that Jay sues Steve in civil court. He claims that he was injured when Steve negligently ran over him while riding his motorcycle on Hillside Avenue on April 24, 2005. Hillside Avenue is a major street in SuccasunnA) From the following facts that arise during the trial, to which is it most appropriate for the judge to apply the Doctrine of Judicial Notice?A) On April 24, 2005 Hillside Avenue was wet, based on the judge's recollection that he had to wear his galoshes to work that day due to the heavy rain.B) Hillside Avenue runs in a north-south direction, based on information generally known by residents of SuccasunnA)C) The brakes on Steve's motorcycle did not work, based on the testimony of Steve's mechanic and an expert witness who is an engineer.D) Steve was speeding at the time of the accident, based on the testimony of a police officer, 2 witnesses and skid mark testing. This testimony was in controversy based upon the testimony of Elaine, a passenger in Steve's car.
    The fact that Hillside Avenue is a north/south running street is the most appropriate item for the judge to take judicial notice of, because it is a matter of common knowledge in the community. A judge will generally take judicial notice of an issue not subject to reasonable dispute if it is: 1) generally known within the territorial jurisdiction of the trial court, or 2) capable of accurate and ready determination by sources whose accuracy cannot reasonably be questioneD) In light of this, B is the best answer.A is incorrect. A judge cannot take judicial notice of a fact simply because he has personal knowledge of that fact. To take notice, as set forth above, we must be dealing with a fact known by members of the community.C is incorrect. The fact that Steve's brakes were faulty is subject to a reasonable dispute, since a person of common knowledge would not absolutely know whether the brakes in Steve's car were faulty. In light of this, the judge cannot take judicial notice of the faulty brakes.D is incorrect. Because the speed at which Steve was driving is subject to reasonable dispute, and is not a common knowledge of all in the community at the time of the accident; it is improper to take judicial notice of this fact.
  41. Sybil sues her stepfather for personal injuries caused by his prolonged physical abuses upon Sybil when she was a minor. Sybil calls Officer Cornelia, who would testify that 15 years ago Sybil's stepfather confessed to Cornelia that he did commit all the alleged crimes. Should the trial court admit the police officer's testimony over the stepfather's objection?A) Yes, because past instances of misconduct may be used to impeach a witness.B) Yes, because the stepfather's confession is an admission of a party-opponent.C) No, because the best evidence of a conviction would be the judgment of the court that convicted Sybil's stepfather15 years ago.D) No, because the assault that is the subject of the evidence is more than 10 years olD)
    Here, we have an admission by a party-opponent. Generally, evidence is admissible if it is relevant and, here, the proposed testimony is clearly relevant as it makes it more likely that the defendant committed the acts that Sybil alleges. The testimony is not counter to any rule of evidence and is, therefore, competent. We are not dealing with hearsay, since hearsay is not applicable to a party-opponent.A may be a true statement in that past instances could be used to impeach a witness, but it is the wrong answer as it focuses on impeachment. Here, we are not dealing with impeachment of a witness, making this an incorrect answer.C is incorrect since a Best Evidence Rule application is incorrect for the present facts. The rule requires that when the contents of a writing are sought to be proved, the writing itself should be admitted if it is available. Here, Sybil is not trying to prove that her stepfather has been convicted (thereby, requiring a copy of his conviction), but rather that her stepfather made an admission 15 years ago.D is incorrect. Generally speaking, under the federal rules a conviction older than 10 years cannot be used to impeach a witness. The problem with Answer D is that Sybil's attorney is not trying to impeach Sybil's stepfather, but to prove that her stepfather made an admission. In light of this,Answer D is incorrect
  42. Jamie sues SpadeHardware for a slip and fall. As Jamie was walking down the birdseed aisle, he slipped on a spot where some spilled suet and millet had been mopped up, leaving a damp floor. At trial the issue is: how wet was the floor? It had been mopped 1 hour before Jamie slippeD) SpadeHardware is ready to offer the testimony of Savage, who will testify about an experiment detailing how long a mopped floor takes to dry. Should the court admit this testimony?A) Yes, if a representative of Jamie was there while the experiment was run.B) Yes, as long as SpadeHardware proves that the conditions of the experiment were similar to the conditions when Jamie slipped (i.e., temperature, humidity, etC)).C) Yes, as long as Jamie had a chance to conduct his own experiment.D) Yes, if it is shown that Savage is not an employee of or associated with SpadeHardware.
    Savage's testimony is only applicable if the conditions of his test were similar to the conditions at the time of the accident. Relevance is evaluated based on how similar the test is to the actual case in point. The experiment is quite relevant and admissible if it is performed under the same conditions of temperature, humidity, etC)... If it is vastly different, however, this might not be a relevant experiment.A is incorrect since an adverse party does not need to be present during a test under the federal rules. Note, however, that Jamie's attorney can cross exam Savage to determine if this is, in fact, an applicable test and experiment.C is incorrect for the same reason: Jamie's attorney can cross-examine Savage without the plaintiff's side having conducted their own experiment relating to the floor.D is incorrect. The person conducting the experiment need not be an independent observer. If Savage was, in fact, a SpadeHardware employee, Jamie's attorney could introduce evidence relating to his potential bias.
  43. Patsy, a builder, contracts to build Grand Ole Office for Hank and begins construction by hiring several subcontractors. GutDirt? digs the foundation, ConcreteMe pours the concrete and AnotherSteelman begins erecting steel beams. Ernest, an employee of AnotherSteelman, is killed when a steel beam snaps out from the concrete footing and on kills him. Ernest's family brings suit against all involved parties and each defendant cross-claims for indemnity against each of the other defendants. At trial, AnotherSteelman, calls Minnie, an expert who testifies that she inspected the poured concrete and found it to be of an improper mix. Minnie testifies that the concrete contained an overabundance of sand based on testing done by an outside lab not associated with Minnie. Minnie finds that the cause of failure was due solely to the poor concrete and nothing else. Is Minnie's testimony admissible?A) Yes, if civil engineers in her field rely upon such materials as plans and reports by outside labs and others in reaching conclusions such as his.B) Yes, if she was not professionally negligent in her analysis.C) No, because her opinion relates to an ultimate issue to be determined in the case.D) No, because her opinion was based upon facts not personally within her knowledge.
    Relevant opinion testimony from an expert witness is admissible, since an expert can base her opinion on facts not known solely to herself, but which are based on facts determined by other experts. Such facts need not be admissible in evidence as long as the facts are of a kind reasonably relied upon by experts in the particular fielD) In light of this, Answer D is clearly incorrectB is incorrect. Whether this analysis is negligent is irrelevant. When the plaintiff has a chance to cross-examine Minnie, they can bring out any cases of negligence in her analysis.C is incorrect. The federal rules allow the trier of fact Qudge) to oversee the admittance of testimony relating to the ultimate issue of the case.
  44. Dumaine owns a bakery, and is charged with murdering Hansel, his employee, by putting him in the baking oven with some muffins. The prosecution seeks to introduce the testimony of Oliver, another employee of Dumaine. Oliver will testify that he was using Dumaine's computer to buy some Canadian prescription drugs online, when he received an instant message from an unidentified person that said, "Hey Dumaine, I hear that Hansel is going to testify about all the drugs that you have been selling out of the bakery. What are you going to do about him?" Assume that the judge allows Oliver's testimony to be admitteD) Dumaine seeks to testify that the "drugs" Oliver testified to hearing about were, in fact, related to the sale of over-the-counter drugs for dogs, such as heartworm and flee pills. The prosecution objects to this testimony, claiming that it is totally irrelevant to the case at hanD) How should the judge rule?A) Admissible, as Dumaine's testimony is relevant in determining whether Dumaine had a motive to kill Hansel.B) Admissible, as it serves to establish the credibility (or lack thereof) of Oliver.C) Inadmissible, since this testimony impeaches Oliver's testimony as related to an irrelevant collateral matter.D) Inadmissible, since the probative value of introducing Dumaine's testimony is greatly outweighed by the danger of unfair prejudice or confusion of issues.
    The testimony proposed is clearly relevant as evidence that there were no illegal drug sales to be covered up, which might have given Dumaine a motive to kill Hansel.B and C are both wrong as the introduction of this evidence has no end results that serve to effect Oliver's credibility.D is a generally bad answer for a number of reasons. First, the objection here is based on relevance and the test set forth is not related to relevance (Rule 403 test). Additionally, even if the recited test were applied, it is highly unlikely that the evidence would be rejected under that test.
  45. James drives up on the sidewalk and runs over Aaron. Before the ambulance arrives James says, "It was all my fault. I'm SO sorry I ran the red light?' (Statement 1) and "I'll pay for all your medical expenses." (Statement 2) Aaron sues and at trial wishes to introduce both statements. The court should rule:A) Both Statement 1 and Statement 2 are admissible.B) Statement 1 is admissible; Statement 2 is inadmissible.C) Statement 1 is inadmissible; Statement 2 is admissible.D) Neither Statement 1, nor Statement 2 are admissible.
    The court should rule that Statement 1 ("It was all my fault I'm SO sorry I ran the red llghtr) is admissible as an admission by a party-opponent and Statement 2 ('77/ pay for all your medical expenses") is inadmissible as an offer to pay medical expenses. By definition, an admission is a statement made, or an act committed, that amounts to a prior acknowledgment by one of the parties to an action of one of the relevant facts. James acknowledged running the red light (Statement 1) and that the injuries were his fault. This is a clear acknowledgment of relevant facts and is, therefore, deemed an admission by a party-opponent. Under the federal rule, a statement to pay a parties medical bills is not admissible as an admission of liability for the injuries, since the payment offer may simply be because James is a nice guy or otherwise.A, C and D are all subsequently wrong because, as explained above, Statement 1 is admissible and Statement 2 is inadmissible.
  46. Clyde is charged with robbing a bank, but in the federal court case alleges that he was vacationing 5 states away in Wisconsin when the robbery occurreD) Clyde calls Bonnie as an alibi to testify that she was traveling on vacation with Clyde at the time in question. When questioned where she was during the robbery and with whom, Bonnie says, "Yes, I definitely know that I was at the Grizzard Motel the weekend of the robbery, because that is the same weekend as my birthday and I was there to celebrate. The thing is, there's no way I was there with a guy as ugly as Clyde." Clyde's attorney seeks to introduce evidence that Bonnie was at the Grizzard Motel with Clyde. Clyde testified to that fact in the preliminary hearing. Clyde's attorney seeks to introduce testimonial evidence from the preliminary hearing. The prosecution objects. The court will find the transcript to be:A) Admissible, but only to impeach.B) Admissible, for both impeachment and as an substantive evidence.C) Inadmissible, because it is hearsay.D) Inadmissible, because the witness is available.
    A hearsay exception exists for any prior inconsistent statement made by a declarant in a situation where the declarant is either under oath, or under penalty of perjury if she lies. A preliminary hearing is such a situation. Therefore, the evidence may be admitted, both to impeach as well as to offer substantial evidence in Clyde's defense. This correct answer then makes, Answer A and C incorrect.D is wrong as it offers an answer that only applies to a hearsay situation, namely, the unavailability of a witness. As set forth in the correct answer, this statement falls within a hearsay exception.
  47. Nicole is charged with conspiracy to embezzle $1 million form her employer, a venture capital firm. At trial, the prosecutor calls Nicole's husband Kaleem to the stand, and asks him to testify about a meeting between Nicole and Tom that Kaleem witnessed on the weekend before their wedding. Which rule governs Kaleem's testimony?A) The choice is Kaleem's.B) The choice is Nicole's.C) Kaleem can testify only if he and Nicole agree that it's okay.D) Kaleem may be required to testify even if he and Nicole object.
    Here, the testimonial privilege belonging to the spouse of a witness, wherein the spouse can prevent testimony of another, would be applicable. Since we are not dealing with a privileged communication between husband and wife, the choice to testify is up to Kaleem and Answers B, C, and D are, therefore, incorrect.
  48. Nina is charged with assault with a deadly weapon. The victim testifies that Nina did it, and Nina's attorney calls Gabriel who testifies that Nina did not attack the victim without provocation. Gabriel states that the victim had been taunting Nina, then pulled a gun on her and shot at her twice, missing both times. Only then did Nina attack in response. Nina's attorney then calls Cruz. He testifies that Nina has a great reputation in the community as a peaceful, kind, honest and loyal person. The victim's attorney - i.e., the prosecutor, moves in limine to exclude the testimony. How should the court rule?A) For the victim, since Nina's attorney cannot introduce character evidence to prove that she acted in conformity with her character.B) For the victim, since Cruz's testimony is totally irrelevant.C) For Nina, since a criminal defendant can put her character in issue.D) For Nina, since a reputation for honesty and loyalty in a criminal defendant is always in issue.
    Here, Nina never testified, and she is at trial for a crime of violence. Her honesty and loyalty traits are not at issue and cannot (and should not) be introduced, since those are completely irrelevant. Therefore, C is incorrect.A criminal defendant may offer evidence of character relevant to the charges, and so Answer A is wrong.D is a misstatement of law.
  49. Ducky is driving a truck for ConvoyTrucking, when he runs over Kris and injures him. Kris sues, alleging that Ducky was drunk at the time of the accident, and that ConvoyTrucking was negligent in hiring Ducky, who had a history of drunk driving. The attorneys for ConvoyTrucking call Ducky as a witness, asking him to testify that he was not drunk. On direct examination, Ducky states that he had drunk 8 beers during lunch as part of his new "liquid diet," and that he was "totally stoned" when he hit Kris. The attorneys then seek to confront Ducky with the testimony he gave during the deposition in which he said, "I was flat sober. It was just an accident." Will the court allow it?A) No, the statement is hearsay not within any recognized exception.B) No, ConvoyTrucking cannot impeach it's own witness.C) Yes, but it may be used only to refresh Ducky's recollection of his prior testimony.D) Yes, it can be used to impeach and also as substantive evidence that Ducky was, in fact, sober.
    Under the Federal Rules, this prior inconsistent statement originally offered by Ducky can be used to impeach him and may further be used as substantive evidence. A prior statement made under oath during a deposition will not be deemed hearsay. Therefore, A is incorrect.B is incorrect because the Federal Rules permit a party to impeach its own witness, even if not "surpriseD)"C is incorrect because it is too narrow. ConvoyTrucking can use the testimony to impeach and also as substantive evidence.
  50. Bear is a mechanic at a dealership who often test drives the cars that he fixes to be sure that everything works. One day, after fixing a LeDorien, Bear gets into a huge accident while speeding and showing off for some girls. The accident is with Boyd, who failed to stop at a red light. Boyd is thrown out of his car and lands in a pen of pigs. His car is totaleD) Bear realizes that Boyd is really hurt and runs to the pig pen to help him. Bear apologizes profusely saying, 7 did a rush job on this LeDorian and didn't fix the brakes as well as I usually do. I am SO sorry." After the accident Bear readjusts the brakes. Boyd brings an action for damages against the dealership where Bear works. Assume that Boyd's attorney introduces evidence that Bear readjusted the brakes on the car after accident. The dealership's attorney objects. How should the court rule?A) Overruled, because it tends to prove negligence.B) Overruled, because it is relevant to Bear's state of minD)C) Sustained, because it is an assertive conduct.D) Sustained, for public policy reasons.
    Evidence of repairs made after an accident is inadmissible to prove negligence. This is for public policy reasons, such as encouraging a person to fix a lingering problem on their car and hence prevent further possible accidents. Boyd's attorney is not permitted to introduce such evidence to prove negligence.A is wrong in light of the correct answer, as subsequent repairs cannot be used to show Bear's negligence.B is wrong because Bear's state of mind is not in issue. Additionally, the intended evidence does nothing to prove anything related to Bear's state of mind at the time of the accident.C is wrong. Even if the brake adjustment was deemed an assertive conduct, it is not hearsay if it is viewed as a statement and is, therefore, an admission by the agent of a party-opponent. Additionally, it is unlikely the court will deem the readjustment of the brakes assertive conduct, as assertive conduct is conduct intended to be a substitute for words. Here, Bear is not trying to convey anything in lieu of a statement by readjusting the brakes.
  51. Zeus is tried for the rape of LedA) Zeus calls Helen, who lives next door to LedA) At the time of the rape, Helen was outside washing her car and saw the perpetrator run form Leda's apartment. Helen further identifies that the person she saw was a stocky 5 foot tall person with white feathered hair and clearly not the svelte 7 foot tall brunette Zeus, who has been charged with the crime. During the prosecutor's cross-examination, which of the following defense objections would most likely be sustained by the judge?A) "Helen, weren't you convicted of perjury 15 years ago?"B) "Weren't you smoking crack while washing your car and totally out of it?"C) "Helen, haven't you and Zeus been best friends since kindergarten?'D) "Weren't you fired from your job last week at the bank for embezzling funds from little old ladies?"
    Under the federal rules of evidence a prosecutor cannot inquire into a crime or prior conviction that relates to dishonesty, if it is more that 10 years old, as a court deems this too remote. In light of this, A is correct: an objection to this question would be sustaineD)B is permissible. It relates to Helen's ability to perceive what was going on and would be a legitimate question on cross-examination.C shows a possible bias on the part of Helen.Note: Bias is a basis for impeachment of a witness. Therefore, this question is permissibleD relates to a prior bad act that shows dishonesty in Helen. Such acts may be asked about on cross-examination of the witness.
  52. Suppose the police set up a stolen property sting where they sell stolen property to, and buy stolen property from, anyone who comes into their Newark warehouse. They film everyone coming and going with hidden cameras. Sumner is brought up on charges for receiving stolen property in connection with his arrest by the undercover police. The prosecution seeks to introduce evidence that Sumner entered the Newark warehouse, offering to sell a 50 inch plasma television to one of the undercover police officers. Assume that the evidence is held to be inadmissible. The most likely reason is that:A) It is hearsay not within any exception.B) It violates Sumner's privilege against self-incrimination.C) A proper foundation was not established for its introduction into evidence.D) Criminality may not be proven by specific instances of misconduct.
    When dealing with video evidence, it is necessary to first properly authenticate the evidence such that it is admissible. Absent a proper foundation, this evidence cannot be admitted into trial.A is wrong because the tape constitutes an admission and, therefore, is not hearsay.B is wrong. The privilege against self-incrimination can only be applied to the testimony of a person. The police film is not testimony.D is a bogus answer that appears to be based on nothing concrete. Note: AVOID THESE ANSWERS LIKE THE PLAGUE.
  53. McFly is charged with killing Biff in the conservatory with a candlestick on January 1st. While at trial, McFly calls Brown to testify that McFly has a reputation within the community as a Ghandi-like peace loving man who will not even kill an insect. A court will find this testimony:A) Admissible as tending to prove that McFLy is innocent.B) Admissible as tending to prove that McFly is a believable person.C) Inadmissible, because reputation is not a proper way to prove character.D) Inadmissible, because McFly has yet to take the stanD)
    When dealing with a case of a defendant proving his innocence, the defendant is free to introduce testimony relating to his good character tied to the trait involved in the case (pacifism vs. the alleged murder). Evidence such as this, is not restricted to impeachment purposes only. Therefore, B and D are incorrect.Additionally, under the federal rules the witness (Brown) can both testify to the defendant's reputation for the trait in question (pacifism) as well as give his opinion of the defendant, making Answer C incorrect.
  54. Steve is charged with killing Russell by throwing him into a furnace on October 9th. Steve's attorney calls Matt, Steve's neighbor, to testify that on October 7th Steve asked Matt to pick up his newspaper since he was going on a two week trip to visit the Dalai Llama in India, which was 7,000 miles from Steve's home in New York. Matt's testimony will likely be held:A) Admissible, because it is not hearsay.B) Admissible, because it is a declaration of Steve's mental state.C) Inadmissible, because it is hearsay not within any exception.D) Inadmissible, because it is irrelevant.
    Prior case law states that in a situation such as this, the defendant's state of mind is admissible into evidence to show that the declarant (Steve) acted in conformity with his expressed declaration.A is wrong since this is hearsay. Note, however, that this falls within a hearsay exception, as set forth in the correct answer.C is wrong. While we are dealing with hearsay, we are further within a hearsay exception.D is wrong. We are dealing with Steve's alibi, which is highly relevant.Note: An alibi will always be relevant, and, therefore, there is no basis for denying theintroduction of Steve's alibi.
  55. Bill and his son, Theo, are driving home from a baseball game when Tony crashes into them. This occurs at an intersection where all junctions have a stop sign. Bill and Theo sue Tony. At trial, Bill's attorney calls Theo to testify that while sitting in the front seat of his dad's car, he saw Tony's big blue van and saw that Tony never even slowed, let alone stopped, for the intersection. Assume that Tony's attorney submits that Theo is not competent to testify. Should Theo's testimony be admitted?A) Since Theo is a close relative of Bill, Theo cannot testify on Bill's behalf.B) No, Theo is a party-plaintiff and cannot, therefore, testify to the facts of the accident.C) Yes, assuming that Theo is at least 12 years olD)D) Yes, there are no facts to indicate that Theo is incompetent to testify.
    Note that we have no facts indicating that Theo is incompetent to testify. A witness will be deemed incompetent based on age or mental incapacity, but both are viewed in light of the ability of the witness to understand the oath and the need to testify truthfully. Therefore, A and B are incorrect. Answer C is further wrong since we have no predefined age at which a person is deemed competent or incompetent. It is based solely on the ability to understand the oath and the need to testify truthfully.
  56. Adam sues Drew for breach of contract, wherein the terms of the contract are in dispute as well as the existence of the contract altogether. Adam's attorney calls Adam to the stand and seeks to get him to testify that a month ago Adam and Drew met at a diner and reached an agreement that they subsequently reduced to writing. Adam intends to state, "The contract was accidentally destroyed when the waitress dropped a pot of coffee on it. But I swear Drew had signed on to buy 1 plasma TV per month for the next year from me at a price of $2,000 each." The quoted testimony is admissible only if:A) The judge finds that the writing is, in fact, unavailable.B) The judge finds that Adam is accurately relating the terms of the contract described in the writing.C) The jury finds that the writing is unavailable.D) The jury finds that Adam is accurately representing what was in the contract.
    A judge must determine that the document is unavailable in order for the testimony to be admissible, as the contents of a document are sought to be proven and the document is not collateral.B is wrong because, as stated, the document itself must be produced unless it is not available.C and D are wrong because the judge, not the jury, makes these types of determinations.
  57. GlamDiva is injured when her blinged-out tour bus is t-boned by a truck full of bowling balls. It was GlamDiva's bus driver that ran a stop sign and caused the accident. Once the bus stopped rolling David, a fellow passenger, called out, "Damn man, we had to be doing at least 80! This bus driver was flying!!! Is everyone okay?" GlamDiva sues Van Cool (the tour bus rental company) for her injuries. She sues on the theory of respondeat superior and negligent hiring. GlamDiva's attorney calls David to testify about his statement. Is his testimony admissible?A) Yes, if it is offered as a prior consistent statement as to David's testimony at trial.B) Yes, because the statement was an excited utterance.C) No, because the statement is hearsay that does not fall within one of the exceptions.D) No, because the testimony is improper opinion evidence.
    The question tips us off that this may, in fact, be an executed utterance due to the exclamation points. If David was under the excitement and stress of the situation, his testimony is allowed to come in because an excited utterance is an exception to the hearsay rule. In light of this, Answer C is clearly wrong.A is wrong since one can only introduce a prior inconsistent statement to rebut a charge that a person is lying. Here, we have no reason to believe that David is lying for any reason.D is wrong. A lay witness opinion can, in fact, be admitted if based on the perception of the witness and NOT based onscientific or expert knowledge.
  58. Which of the following questions is likely to be found a "leading" question?A) On direct examination, counsel asks his own client: "Your address is 381 Highland Avenue, correct?'B) On cross-examination of an expert witness the attorney says, "Isn't it true that most intellectual property attorneys are the highest paid in the attorney ranks?'C) On direct examination of a 5 year old child, the prosecution asks, "Did the defendant touch you there?'D) On direct examination of a witness, the plaintiff attorney asks, "Was the light red when Harry blew through the intersection in his yellow sports car blasting country music?'
    D is the correct answer, as questions A, B and C are allowable based on their situations. Question A deals with a preliminary matter that is not in dispute. Question B takes place on cross-examination. Question C is during the examination of a young chilD) In each of these situations, guiding questions are generally permissible. On direct examination of a non-interested witness, however, guiding questions are not permitted, making Question D "leading."
  59. Russert brings suit against Donald, the owner of ApocalypseExtermination, on the basis that Donald's actions have resulted in deadly fumes filling up his house. While Russert is holed up in a bug infested mote, Donald calls and says, "If you have any medical bills, ApocalypseExtermination will pay We are committed to the community and would never intentionally hurt a member of it We are sorry about filling your house with poisonous gas as you slept and for causing your illness." Russert's attorney seeks to introduce this voicemail at trial. ApocalypseExtermination's attorney motions to strike and the court does so. Was the court's action correct?A) Yes, because the testimony relates to inadmissible hearsay.B) Yes, because the statement was made in connection with an offer to pay medical expenses.C) No, because the statement includes an admission by a party-opponent that they were negligent.D) No, because the statement is a factual admission made in connection with an offer to compromise.
    Under the federal rules an offer to pay medial bills cannot be introduced into testimony, but an admission by a party-opponent associated with this offer is, in fact, admissible. Here, we clearly have an admission by Donald and this admission can be introduceD) In light of this, Answer B is wrong.A is wrong because the statement is an admission, and all admissions are deemed NOT to be hearsay.D is wrong because we have no applicable offer to compromise. Donald's message concerning ApocalypseExtermination simply states that they will pay all bills, and seek nothing in exchange.
  60. Choy is a minor and, with his parents, sues Cristos for $100,000 in injuries that Choy sustained when Cristos ran over him one evening while delivering pizzas and rushing around to get them delivered in less than 3 hours. Choy was knocked to the sidewalk from his bike and rendered unconscious. Cristos claims that it was not he who hit Choy. Aside from damages, the real issue is whether, in fact, it was actually Cristos who hit Choy. While questioning Cristos, Choy's attorney asks, "Isn't It true that you put a bigger engine in that Honota of yours for $1,000 so that it would go faster?" The court should rule this question to be:A) Objectionable, because it is leading.B) Objectionable, because it is irrelevant.C) Unobjectionable, because it shows that Cristos was an irresponsible pizza boy.D) Unobjectionable, because Cristos can testify as to what he spent on upgrading his car, even if there is a receipt form a mechanic that details everything.
    Relevance is based on whether evidence helps decide an issue in dispute. The issue in dispute, here, is whether Cristos ran over Choy, and the bigger engine in Cristos' car will likely be found irrelevant. While Answer B may seem questionable as a 100% correct answer, it remains the least objectionable.A is wrong. Had the question been asked by Cristos' attorney on cross-examination it would not be leading. Choy's attorney asking on direct examination, however, would clearly be considered leading the witness.C is wrong. Evidence of a character trait, such as an irresponsible nature, would be inadmissible.D is wrong. The question at hand concerns whether the evidence is relevant and Answer D addresses whether this evidence should be under the Best Evidence Rule. Note: When choosing an answer, be sure to choose one that answers the question askeD)
  61. Tracy dies without a will, leaving her only son, Meachum, as her heir. During probate, Nick, Tracy's "personal assistant," makes a claim for the house, and produces a deed in which the property was allegedly transferred to him 2 months before Tracy died of a stroke, which was suffered while in the heat of passion. Meachum disputes the claim, arguing that the signature on the deed is fake. Assume that this all takes place in front of a jury and that Nick testifies that he had an agreement with Tracy. In exchange for giving her a sponge bath each evening, Tracy promised to turn over the house when she dieD) Meachum's attorney objects to the evidence. Which is the best reason for the judge to rule for Meachum and not allow the evidence to come in?A) It is hearsay.B) A contract is not a proper will.C) The agreement violates the Statute of Frauds.D) The evidence violates the Best Evidence Rule.
    Since Nick testified that there was a written agreement in effect, in order to prove the terms of that agreement the Best Evidence Rule states that Nick must either produce the written agreement or somehow explain the absence of the agreement. Nick has done neither.A is wrong. This out-of-court agreement can be found to be a "legally operative fact," thereby making it NOT hearsay.B is wrong because Nick never mentions that this is a will. He just mentions that he had an agreement.C is wrong. Because Nick is testifying that there was a written agreement, a Statute of Frauds violation is not pertinent, as the Statute of Frauds concerns contacts not reduced to writing.
  62. Kathy is on trial for killing her boyfriend Lee by feeding him into a wood chipper and directing the exhaust of the chipper into a salmon stock pond where the salmon could eat Lee's body and grow into strong steelheads. Kathy claims that Lee jumped into the chipper to commit suicide. Kathy takes the stand and on cross-examination the prosecutor asks, 'Were you convicted of robbing a bank 2 years ago?" How should the judge rule on this question?A) Proper, since upon taking the stand Kathywaived the privileged against self incrimination.B) Proper, since a matter such as this one (a matter having bearing on Kathy's credibility) may only be asked about by a prosecutor.C) Improper, because the purpose of the question was to elicit prejudicial evidence.D) Improper, because on cross-examination the prosecutor is limited to matters testified to on direct examination.
    Once a defendant takes the stand, they put their credibility in issue. During an impeachment procedure by the prosecution, any instances of prior felony convictions can be addressed in an effort to impeach the witness.A is wrong. The question asks about how the judge should rule, not on whether Kathy is obligated to answer.Note: Be sure to answer the question askeD) Don't be tricked into choosing correct answers to other questions.C is wrong. Numerous questions asked by the prosecution will be deemed "prejudicial" from the defendant's point of view.D is wrong. A party is not prohibited on cross-examination from inquiring into a witness's credibility.
  63. Louis is helping Emmett change his oil. As Emmett is jacking up the car, the jack collapses crushing Louis under the engine block. Louis's wife sues Emmett for wrongful death for $10 million dollars. Louis's wife is called as a witness to testify that Louis was busy loosening the drain plug and did not realize that Emmett had tripped on the jack causing the car to fall on Louis. As a witness, Louis's wife is:A) Competent, as long as she is testifying as the personal representative of her husband's estate.B) Competent, in spite of the fact that she is the plaintiff.C) Incompetent, as she has no qualifications that allow her to give opinion evidence.D) Incompetent, because she cannot testify for her husband in a civil case.
    A person who has an interest in the outcome of a suit is not instantly disqualified from testifying. They may serve as witness in a trial and do not need to be acting in the role of personal representative of the deceased's estate. This makes Answer A incorrect.C is wrong since Louis's wife is testifying to what she saw, not an opinion.D is wrong because the theory of common law spousal incapacity is no longer applicable.
  64. Lynnell, Elaine, Chris and Zeke are driving to Graceland to meet Elvis in a diner when out of nowhere they are run off the road and over a cliff by a King Delivery Co. truck driven by Aaron. Though Aaron was driving the truck, he had a buddy named Buddy along for the ride. Lynnell, Elaine, Chris and Zeke sue the King Delivery Co. and Aaron. Their attorneys allege that the King Delivery Co. was negligent since Aaron had been awake for 36 hours straight on a trip from California to GracelanD) The King Delivery Co. answers the complaint by claiming that Aaron was actually high on psychedelic drugs and cocaine when the accident occurreD) They seek to introduce the testimony of Officer Priscilla, a Tennessee Highway Patrol officer who overheard Aaron say just after the accident, "Awe damn, dude. Buddy, this was probably my fault since I'm all drugged up on blow and shroomsr Assume that the court takes official notice that "shrooms" are psychedelic mushrooms and "blow" is cocaine. This evidence is likely to be found:A) Admissible, as an admission of a party.B) Admissible, since this same statement could come in as part of Officer Priscilla's accident report.C) Inadmissible, if Aaron was "drugged up" he could not have been making a valid assertion as to fault.D) Inadmissible, because it is hearsay that is not within any exception.
    This is not hearsay as it is an admission by a party-opponent, and the admission of a party-opponent, namely, a party's own statement offered against him, is admissible. As this statement is admissible and not hearsay, Answer D is obviously wrong, wrong, wrong.B fails to offer a valid way by which the statement could come in. Arguments based on speedy trail or expediting the process are usually wrong.C is wrong since having a rational basis for a statement is not a prerequisite. If the statement is made by a party, and is relevant and offered by the opponent to the party making the statement; it will come in as a non-hearsay statement.
  65. Dragon is tried for assault after allegedly stabbing Iggy. Dragon represents himself and calls Sooki to the stand, and asks"Sooki, are you familiar with my rep in the town?' Sooki says, "Yep." The prosecution does not object. Dragon than asks, "And what, Sooki, is my rep in town regarding my peaceful behavior?" At which point, the prosecutor objects. The testimony is:A) Inadmissible, because reputation must be proven by generalities and not by specific items of character.B) Inadmissible, because Dragon failed to take the stand C) Admissible, as this tends to prove that Dragon's reputation does not match with the criminal charge against him.D) Admissible, under the general rule that the defendant is given the benefit of the doubt when testimony is questionable.
    When dealing with a criminal trial, the defendant is always free to introduce evidence relating to his good character. This is valid regardless of whether the defendant takes the stand, making Answer B incorrect.A is wrong since the defendant's evidence of good character can only be established by reputation evidence and not by instances and fact. Here, the defendant does, in fact, provide the proper reputation instance and not specific facts.D is wrong. There is no such rule and the testimony is not questionable.
  66. Dragon is tried for assault after allegedly stabbing Iggy. Dragon represents himself and calls Sooki to the stand, and asks, "Sooki, are you familiar with my rep in the town?' Sooki says, "Yep." The prosecution does not object. Dragon than asks, "And what, Sooki, is my rep in town regarding my peaceful behavior?" At which point, the prosecutor objects. Dragon then calls Zed to testify as an alibi witness on his behalf. Zed states that he knew that Dragon was, in fact, visiting his sister in California, 2000 miles away. The prosecution objects to this statement and then asks Zed, "Is Dragon related to you?' How should the court rule on this question?A) Proper, because it relates to bias.B) Proper, because relatives are not competent witnesses.C) Improper, because the question goes beyond the scope of direct examination.D) Improper, because it involves a collateral matter.
    On cross-examination, a person is always allowed to attempt to impeach the witness. One means by which to impeach a witness is to show bias. By asking about a familial relationship, the prosecutor is essentially asking whether Zed is, in fact, biaseD)B is wrong. A relative can clearly testify in a case such as this, as long as the relative (i.e., the witness) has personal knowledge and declares that they will testify truthfully.C is wrong. While attempting to impeach a witness, a party can go beyond the scope of the direct examination.D is wrong. When dealing with a potential bias charge, evidence that comes about during cross-examination is never deemed to be collateral in nature.
  67. A bank is robbed on a Monday morning by a man dressed as a leprechaun. The man is 3 foot 6 inches tall. On Tuesday, the police get a tip from Jon saying that Jake, a person also of short stature, has treated the entire condo complex to drinks at a local bar, bragging that he just struck it rich and flashing a briefcase full of $100 bills. The police go to the condo, knock and Jake answers. The police ask about the rolls of hundreds and Jake states that an old friend of his paid off an old debt regarding a bet they haD) The police leave, only to be called back by the custodian who states that he found a green leprechaun costume and fake beard in the recycle bin. The police return and arrest Jake. After obtaining a search warrant, they search his condo only to find nothing. At trial, the prosecution seeks to introduce evidence regarding the leprechaun outfit. The outfit evidence is:A) Admissible, as circumstantial evidence that Jake committed the robbery.B) Admissible, as direct evidence that Jake committed the robbery.C) Inadmissible, because it is irrelevant.D) Inadmissible, unless the prosecution proves beyond a reasonable doubt that it is the exact same clothing worn by the robber.
    We clearly have circumstantial evidence and are asked to draw an inference that the costume and beard belong to Jake, who is also a Little Person like the bank robber, based on the proven fact that the clothes were found in a dumpster at his condo complex.B is incorrect. Direct evidence is evidence that when believed by a jury, serves to establish by itself one of the questions of the case without the need for an inference.C is incorrect. Finding a costume in Jake's condo complex, similar to the one used in the robbery, serves to make it more probable that he was responsible for the crime.D is incorrect for misstating the applicable standarD) Answer D states the standard for proving guilt, not the standard regarding the admissibility of evidence.
  68. Bear is a mechanic at a dealership who often test drives the cars that he fixes to be sure that everything works. One day, after fixing a LeDorien, Bear gets into a huge accident while speeding and showing off for some girls. The accident is with Boyd, who failed to stop at a red light. Boyd is thrown out of his car and lands in a pen of pigs. His car is totaleD) Bear realizes that Boyd is really hurt and runs to the pig pen to help him. Bear apologizes profusely saying, 7 did a rush job on this LeDorian and didn't fix the brakes as well as I usually do. I am SO sorry." After the accident, Bear readjusts the brakes. Boyd brings an action against the dealership where Bear works for damages. Assume that Boyd's attorney introduces evidence that Bear readjusted brakes after the accident. Bear's attorney counterclaims on the basis that Boyd's negligence in running the red light caused the accident. Bear's attorney calls Fisher, Boyd's friend, to testify that Boyd does not believe in red lights and runs them all. Boyd's attorney objects to this. How should the court rule?A) Sustained, because it is not the best evidence.B) Sustained, because character evidence is inadmissible in a civil case.C) Overruled, because it is evidence of habit.D) Overruled, because it is self-serving.
    Here, Fisher's evidence relates to habit and is admissible. Habit evidence describes a person's action when faced with a similar set of circumstances. Evidence of habit can be used to prove that a person acted in accordance with their habit. In light of this, Fisher's testimony relating to Boyd's habit of running red lights is admissible and the objection should be overruleD)A is incorrect. The Best Evidence Rule is inapplicable, as it relates to a situation where the terms of a writing are material and require that one must provide the writing. Here, we have no writing and, therefore, no application of the Best Evidence Rule.B is incorrect since the proposed testimony is not character evidence, but rather relates to habit. Character evidence relates to a persons traits or ordinary behavior. Habit evidence, as we have here, relates to a person's continual performance when faced with certain situations (e.g., red lights at intersections) and is admissible.D is incorrect. The fact that the objection is self-serving is meaningless. When viewed from afar, all objections are self-serving to the party making them.
  69. Chad sues Clarita for injuries he suffered while working for him as a cesspool cleaner. Clarita files a cross complaint. It states that Chad was negligent when he cleaned the cesspool with gasoline and continued to smoke his cigarette while entering the cesspool, thereby, causing the explosion that both damaged the drain and injured ChaD) Chad denies these allegations. Clarita's attorney calls Lulu, another employee, to the stanD) Lulu was also in the cesspool with Chad and testifies that as she pulled Chad from the burning sewer he was screaming, "&*%$ me! I'm such a frickin' idiot! I should never have brought my cig down there. What was I thinking?' Chad's attorney objects to this testimony. The trial judge should rule that the testimony is:A) Inadmissible, because it is hearsay not within any exception.B) Inadmissible, because it is improper opinion evidence.C) Admissible, as an admission.D) Admissible, as being a prior inconsistent statement.
    Here, A is the only answer that can be viewed as correct. In light of this, it is easier to explain the wrong answers first.B is incorrect since the opinion here was based on Lulu's perception and her opinion would, in fact, be helpful to determine if Chad was negligent in his actions.C is wrong because Lulu is not a party to the present case. Admissions only relate to parties in the case, not outsiders like Lulu.D is wrong. We have no indication that Chad has testified, therefore, we cannotview this statement as a prior inconsistent statement. Here, we have a statement that is clearly hearsay, but could potentially fall within the excited utterance exception. There is, however, no answer offering excited utterance as a choice. Because of this, you must go with A as the correct answer.
  70. Donald sues Swish, a pharmaceutical company. He alleges that after taking some medication for his erectile dysfunction, a blood clot formed in his leg, requiring that it be amputateD) Swish retorts that Donald's claim is bogus and that there is nothing in their drug that could result in a blood clot. Donald's attorney calls Dr. Patel, an acclaimed surgeon, to testify that in the last year she has treated 10 people taking the same medicine as Donald for blood clots in their legs. Swish's team of attorneys objects to this testimony. The judge should find the testimony:A) Inadmissible, because the testimony of the other patients' conditions is hearsay not within any exception.B) Inadmissible, as it is violative of the Best Evidence Rule.C) Admissible, if Donald's attorney can show a connection between the patients' clots and Swish's medicine.D) Admissible, but only if Dr. Patel is qualified to testify as an expert.
    Dr. Patel's testimony can come in, but Donald's attorney needs to establish a link between the medicine Donald and other patients took, and the subsequent clots that developeD)A is wrong as we are dealing with the perceived opinions of a witness who is on trial and available for further cross-examination.B is wrong. The Best Evidence Rule only applies to a situation in which one party is attempting to prove the contents of a writing. The plaintiff attorney is not trying to do this, but rather is introducing testimony of Dr. Patel regarding her patients. The Best Evidence Rule requires a showing that the patients are unavailable before letting the doctor testify as to their condition.D is wrong since Dr. Patel is not acting as an expert witness, which would require her to be qualified as an expert. Rather, she is providing an opinion testimony.
  71. Samir sues Kylie for breach of contract regarding the purchase of 10 boxes of new Cocoon Digital music players. Samir alleges that Kylie failed to pay, as stated in their contract. Kylie defends this allegation by introducing the fact that Samir only sent a shipment of 6 boxes (not the intended 10) which she rejecteD) Samir's attorney introduces a bill of lading stating that 10 boxes of Cocoons were shipped and the required taxes paid on all 10. Kylie's attorney seeks to introduce a clause from the contract which states, "Kylie has no obligation to accept Cocoons unless delivered 10 boxes at a time as requesteD)" Samir objects. How will the court rule?A) Kylie's attorney is free to introduce evidence such as this in order to make the transaction understooD)B) By not objecting to the contract, Kylie's attorney has waived the right to introduce any part of it.C) Kylie's evidence is inadmissible as irrelevant.D) The contract at issue is hearsay.
    Here, Samir's attorney has introduced a part of the contract, the bill of lading. In light of this, Kylie's attorney is free to either cross-examine regarding this portion of the introduced contract, or in the alternative can introduce additional portions of the writing to make the entire writing understooD) In light of this, A is the correct answer.B is an incorrect statement of the law and is clearly incorrect. Such brazen and broad answers are generally incorrect.C is incorrect. The introduction of this evidence serves to make it more probable that Kylie has, in fact, complied with the contract. Not allowing the introduction of this evidence makes it seem more likely that Kylie has not complieD)D is incorrect since we are not dealing with a situation of hearsay. Here, the contract is a legally operative fact.
  72. Sylvester and Rea collide with each other in their fancy sports cars. Rea is indicted for driving drunk, which has a maximum sentence of 2 years in jail. Blathe, who witnessed the entire accident, testifies in front of the grand jury. Rea pleads guilty and she is fined $500. After completion of the criminal proceedings, Sylvester sues Rea in a negligence suit, seeking personal injury damages. In the negligence suit, Blathe testifies that Rea was driving on the wrong side of the road and weaving back and forth. Rea's attorney seeks to question Blathe about his grand jury trial statement in which he said that Rea was driving just fine. The court should rule this testimony:A) Admissible for impeachment only.B) Admissible as substantive evidence only.C) Admissible for impeachment and as substantive evidence.D) Inadmissible, because it is hearsay.
    The grand jury statement may be admitted for two reasons: to impeach and as substantive evidence. A hearsay exception exists for any prior inconsistent statement made under oath at a prior proceeding or deposition. Impeachment allows one to submit that the witness's credibility is lacking due to statements that are inconsistent with some material part of his present testimony. Generally speaking, statements such as these (i.e., those made by the declarant other than while testifying at the trial or hearing) are generally found to be hearsay if offered to prove the truth of the matter asserted, allowing the statement only to be admitted to impeach. Here, however, Blathe's statement was made under oath at a prior proceeding, and is admissible nonhearsay (i.e., substantive proof of the facts stated).A states an outdated rule of law wherein prior inconsistent statements were limited to impeachment, regardless of the situation under which they were made. This rule has given way to the one explained in the correct answer.B is incorrect since it fails to address the availability of the grand jury statement for impeachment purposes. A prior inconsistent statement may always be used to impeach the credibility of a witness.D is wrong for two reasons. First, a prior inconsistent statement is always admissible to impeach the witness. Second, because the prior inconsistent statement occurred during a grand jury proceeding, it is admissible nonhearsay.
  73. Cass, an expert witness, is called to the stand in a criminal trial. The prosecutor examines Cass for one hour and then the defense cross-examines her for 4 days straight. The prosecutor seeks to have the court terminate the cross-examination. Coincidentally, the prosecutor is leaving on vacation in one day. Can the court approve the prosecutor's motion?A) Yes, unless the court finds Cass's testimony to be relevant.B) Yes, assuming that the court finds that the defendant has had a sufficient opportunity to cross-examine.C) No, assuming that the cross-examination relates to subject matter brought up in direct examination.D) No, when dealing with a criminal trial, due process outweighs the thought of expediting the trial process.
    While a party is always awarded the right to cross-examine a witness, the presiding judge has discretion as to the length of that cross-examination. If the judge feels that an attorney's lengthy examination is no longer properly utilizing the court's time, or if she feels that a witness is being harassed by an attorney, she may terminate continuation of that examination. Note: Cross-examination content is limited to: 1) matters brought out on direct examination or related to direct and 2) matters affecting the credibility of the witness. Whether a line of questioning meets these criteria is also at the discretion of the judge.B is wrong. The court is free to terminate a cross-examination at any time (even if the line of questioning is relevant to the case) in order to avoid wasting the court's time or to avoid harassing the witness.C is incorrect in a similar vein. The court retains the privilege to terminate even if a line of questioning relates to matters brought up on direct. In light of this, it is wrong to say that if the questioning has any relation to the direct examination, then the court cannot cut you off. It can.D is incorrect even though it sounds "fair." Due process only requires that one is awarded the opportunity to cross-examine, not an unlimited opportunity to cross-examine. Let the law be your guide, not fairness-related principles. Due process concerns are clearly important, but assuming that there has been time for some cross-examination and the judge has ruled that the questioning has strayed too far in scope or too long in length, due process has not been infringed upon by a termination.
  74. Strauss sues Morrow and Morrow's employer based on a personal injury claim in which Strauss is hit on the head by a bucket of paint that was dropped by Morrow while she was painting the Golden Strait Bridge. To prove that the paint was dropped by Morrow, Strauss's attorney introduces the now empty and dented paint can. The can has a sticker attached stating the paint color: "Strait Bridge golD)" Strauss's attorney also introduces evidence that the same brand of "Strait Bridge gold" paint was sold to Morrow and her employer for use in painting the bridge. Which best describes this evidence?A) The only relevance here is the name "Strait Bridge gold," and that name is hearsay. The name and evidence must be kept out.B) The can is evidence that Morrow dropped it. It isn't hearsay and is admissible.C) The fact that the can was the same brand and said "Strait Bridge gold" is irrelevant and fails to establish more likely than not that it was Morrow who dropped it.D) The can is clearly relevant circumstantial evidence and not hearsay if offered to prove that Morrow dropped it.
    Relevant evidence is any evidence that tends to make the existence of any fact of consequence to an action more probable than it would be without that evidence. Here, this evidence does just that and is, therefore, relevant. The paint can is circumstantial evidence of fact, as it is used as the basis for an inference that another fact is true; the fact that it is the same brand and color may prove that Morrow dropped it.Note: Direct evidence serves to prove a fact about the object as well as serve as an end in itself (i.e., the fact that the paint can said "Strait Bridge gold" proves that the paint was the same brand). The paint can is not hearsay since hearsay is a statement, other than one made by the declarant while testifying at the trial or hearing, offered in evidence to prove the truth of the matter asserteD) This is not the case.A is incorrect. As set forth above, the paint can is not being introduced to prove that the can was a "Strait Bridge gold" brand can (hearsay), but rather to form the basis for an inference that it was dropped by Morrow. Thus, there is no hearsay problem.B is wrong since the paint can is only circumstantial evidence, rather than direct evidence that it was dropped by Morrow and hit Strauss.C is wrong to conclude that the paint can is irrelevant. The fact that the can is the same "Strait Bridge gold" tends to make it more likely that it was Morrow who dropped it. On this point, the can is clearly relevant. Answer C states an incorrect test for relevance. A relevance test is not a "more likely than not" test. In order to be relevant, there only needs to be a showing that the evidence in question makes things more probable with the evidence than without it.
  75. Barnard sues NoChairLeftBehind, a moving company, after his couch falls off the back of their truck while he is following directly behind them on the highway in his car. Barnard's suit is based on the fact that NoChairLeftBehind was negligent by supplying its driver with a truck that had a broken loading liftgate which carried the couch. He also contends that the driver of the truck was additionally negligent in failing to properly secure the couch in place in lieu of the broken liftgate. Barnard's attorney calls Pammy, who testifies that she used to work for NoChairLeftBehind and knows the driver well. She says that just before the accident she and the driver happened to be talking outside of Barnard's house and that she mentioned to him that the rope holding the couch looked loose. Assume that the defendant objects, this testimony is likely:A) Admissible under an exception to the hearsay rule.B) Admissible nonhearsay.C) Inadmissible hearsay.D) Inadmissible opinion evidence.
    Here, we have admissible nonhearsay. Since Pammy's statement that the couch was loose is not being offered to prove the truth of the matter asserted (that the couch was loose), it is not hearsay. Instead, Barnard's attorney is seeking to introduce Pammy's statement as evidence to illustrate the statement's effect on its recipient. In other words, Barnard's attorney is showing that the driver had notice and is not introducing the statement to show that the rope was, in fact, loose (the matter asserted). In light of this, B is the right answer. Showing such knowledge by the listener is applicable in a negligence case where knowledge is in issue. In light of the correct answer, C is clearly incorrect.A is incorrect. Since this statement comes in because it is nonhearsay, we have no need to look into the hearsay exceptions in order for it to be admitted into evidence.D is wrong on 2 counts. First, the statement is not being introduced to prove the matter asserted (i.e., a hearsay application) but rather to show that driver had notice. This is clearly not opinion testimony. Second, Answer D incorrectly infers that any sort of opinion evidence is inadmissible. This is clearly NOT the case as opinion evidence may be admitted when it is: 1) rationally based on the perception of the witness; 2) helpful to a clear understanding of the witness's testimony or to the determination of a fact in issue; and 3) not based on scientific, technical, or other specialized knowledge.
  76. Nina is beaten and raped by Simon. Nina manages to eventually escape, goes to the police and gives them a photo of Simon that she snapped with her cell phone camerA) The police instantly recognize Simon based on his notorious past convictions and pick him up. Nina is taken to a doctor to be examined that same night and Simon is charged with the attack that same night. At trial, Nina becomes scared and refuses to testify. The prosecution feels that the case is slipping away and puts the doctor who examined Nina on the stand to testify. The doctor will testify that Nina was raped and beaten, and describe what Nina had told him about being raped, beaten etC).. Nina wants to stop the doctor's testimony and invokes the doctor/patient privilege. In which of the following situations is the court least likely to allow Nina to invoke the doctor/patient privilege?A) Nina visited the doctor for diagnosis and treatment and the statements she made to him are only related to this treatment.B) Nina is married to the doctor.C) Nina's statements to the doctor were made public when the doctor made a statement to the police prior to the trial.D) When Nina went to see the doctor she knew that his license had been revoked 20 years ago.
    The doctor/patient privilege does not apply to a situation where the patient knows that the alleged doctor is, in fact, not a doctor at all. The person must be a licensed doctor in order for the privilege to apply. Therefore, in Situation D, no privilege exists at all. Regardless of jurisdiction, a patient needs to relieve a doctor of the doctor/patient confidence in order for them to be allowed to testify.A is incorrect for stating a requirement for the privilege to exist, instead of answering the question. In light of this, A yields an answer that is MOST likely to result in the application of the privilege, not the LEAST likely. READ what they are asking you.B is a totally irrelevant answer. Marriage has no effect on the doctor/patient privilege.C is wrong. The privilege is in the hands of the patient who has control over the privilege. The prior disclosure by the doctor to the police has no bearing on the court's subsequent decision on whether to grant the privilege.
  77. Cami sues Bruce for frauD) She seeks damages following the purchase of a charter fishing business from Bruce. Cami claims that she relied on Bruce's bookkeeping as to the value of the business based on assets and inventory (frozen bait). Cami claims that the bookkeeping turned out to be fraudulent. At trial, Cami's attorney introduces the written contract, without objection from defense, which reads, "HolyMackeral is a solvent company with inventory and assets valued at $1 million." The contract further includes some boilerplate terms like "no other additional or contradictory representations have been made by the seller to the buyer beyond those written above, etC).." Note that Bruce is a lawyer, while Cami is not. Cami's attorney seeks to introduce that during the negotiation process Bruce told Cami that the real value of the business was $2 million, but that for tax purposes they should write the deal up as $1 million. Bruce objects. The evidence should be found:A) Inadmissible, as violative of the Parol Evidence Rule.B) Inadmissible, because it is hearsay.C) Admissible hearsay, regardless of the Parol Evidence Rule.D) Neither hearsay nor violative of the Parol Evidence Rule.
    Cami's testimony does not violate the Parol Evidence rule for hearsay. Hearsay is a statement, other than one made by the declarant while testifying at the trial or hearing, offered in evidence to prove the truth of the matter asserteD) The introduction of Cami's statement is not hearsay as it is not being introduced to prove the matter asserted, but rather to show the effect of the statement on Cami. (Cami relied on the statement when entering the contract for the sale of the boat.) The Parol Evidence Rule states that if an agreement is reduced to a writing, all prior negotiations or agreements are merged into the written agreement and you cannot use extrinsic evidence after the agreement is written. Note however, that the Parol Evidence Rule does not prevent the admission of parol evidence to show that the alleged contract is no contract at all and is, in fact, a void or voidable contract. Parol evidence can be used to disprove a contract based on a claim of fraud, duress, or undue influence which resulted in the consent of the other contracting party. In light of this, parol evidence is permissible in the present case.A is wrong. The Parol Evidence Rule is not violated as explained in the correct answer.B is wrong. The evidence is not hearsay, as it is not being offered for the truth, but rather for a secondary reason as set forth in the correct answer.C is wrong for claiming that we have hearsay. As set forth, this is not a hearsay at all.
  78. Jamal owns a small butcher shop, and is hit by Trish while driving to work. Two months later, he asks his employee, Jane, to go down to the accident scene and take some photos with his digital camerA) Jamal then takes these photos and hands them to his lawyer. His lawyer seeks to introduce the photos into evidence in the suit against Trish. His lawyer also seeks to have Jane testify that she took the photos. He also plans to call Meena, who lives across the street from the accident scene and was the first person to show up. Meena will gladly testify that the scene of the accident was accurately represented by the photos. Whose testimony is necessary to introduce the photos into evidence?A) Jane's testimony is necessary, while Meena's is not.B) Meena's testimony is necessary, while Jane's is not.C) You need both Meena and Jane to testify.D) The picture is inadmissible.
    Admissible photos must be identified by a witness as accurately portraying relevant facts to the case at hanD) A witness familiar with the scene of the crime is sufficient. Generally, it is not necessary to call the photographer of the shots to the stand in order to get a statement that she took the photos. The relevant question here is the actual appearance of the intersection where the accident happened, and Meena is in the best position to answer this.A is wrong. Generally speaking, a photographer's testimony is not necessary to authenticate a photo.C is incorrect because, as stated above, we have no need to call the photographer. D is incorrect because the photograph is admissible if properly identified by MeenA)
  79. Colin is a construction worker who is injured when he is accidentally run over by a bulldozer traveling at 1 mph. His company, MenAtWork Construction Co., pays his medical bills, gives him pay during his recovery, and takes care of his expenses. Colin seeks additional payment because his broken foot has developed arthritis and it is difficult to walk on it when the weather is cold and damp. MenAtWork says that Colin's arthritis has nothing to do with his injury and refuses to pay him his requested disability. Colin sues and his attorney calls his specialist, Dr. Hay, to the stand as an expert witness. Dr. Hay says, "Colin's accident caused his arthritis." On cross-examination, MenAtWork's attorney asks Dr. Hay if he has ever read the treatise "Arthritis as Usual" written by Dr. Cargo. Dr. Hay states that he has read it, but that he did not rely on it. MenAtWork's attorney seeks to introduce a statement from Dr. Cargo's treatise that states "the idea of arthritis being caused by an accident is preposterous. Everyone knows you need years of prolonged medical problems to get it, not a single event." Colin's attorney objects. The court should find the statement from the treatise:A) Admissible, but only to impeach Dr. Hay.B) Admissible, only as substantive evidence.C) Admissible, as both substantive evidence and to impeach Dr. Hay.D) Inadmissible
    The federal rules state that a known treatise can be used both to impeach and as substantive evidence. To impeach, one can cross-examine an expert on statements from a publication that is held as authoritative. Treatises are also admissible as substantive evidence under a federal rule exception to the hearsay rule if: 1) the expert is on the stand and it is called to his attention, and 2) it is established as a reliable authority. Dr. Cargo's treatise is authoritative and MenAtWork's lawyer can use this to attack Dr. Hay's knowledge in the fielD) Additionally, we can read this treatise as substantive evidence because it has been brought to the attention of Dr. Hay and it was he who established it as a reliable authority. Furthermore, it was read into evidence while Dr. Hay was on the stanD)A appears correct as it recites the traditional view, but it fails to include that the federal rules allow the use of a statement in a treatise as substantive evidence.B is incorrect for failing to recognize the ability to use this statement to impeach.D incorrectly infers that the law would not allow introduction of the statement for either impeachment or substantive evidentiary purposes.
  80. Minka is charged with bank robbery. During her prosecution, it is well established that the robber left the bank wearing a dog mask and took off in a getaway car. Bill, Gill and Dan all witness the getaway. Bill is ready to testify that when Minka jumped into the car someone yelled "Get the plate number./" and Bill replied, 7 got it! It is 1234567P' Bill's testimony is:A) Admissible only if Bill does not testify or refuses to testify since his personal testimony would be the best evidence.B) Admissible hearsayC) Inadmissible hearsayD) Inadmissible, because there is no proper foundation or identification of the hearsay declarant.
    Bill's is an excited utterance, a hearsay exception. Excited utterance requires that a declaration made during or soon after a startling event is admissible if the declaration was made under the stress or excitement of the startling event. Here, we are dealing with an out-of-court statement offered to prove the truth of that which is asserted (hearsay), but it falls within an hearsay exception.A is wrong for misapplying the Best Evidence Rule. Best Evidence only relates to a situation where a party is trying to prove a writing. Here, we have no writing, just oral testimony.C is wrong. This may be hearsay on its face, but it is within an exception and in turn is admissible.D is wrong since we don't need any foundation to be laid in the present case.
  81. Otto, a minor, is run over by Wiggum. Otto's father takes Otto to see Bernie, an attorney, intending to sue Wiggum. Bernie, Otto and Otto's father discuss Otto's injuries and the incident. Bernie then says, "You know what? You would be better off with a personal injury lawyer than with me. I really only do patent law." Otto and his father hire another lawyer later that week. The defense believes that Otto is not really injured, and only acting hurt at the urging of his father to make money. The defense subpoenas Bernie to a deposition. He is asked about the seriousness of Otto's injuries. Can Bernie rely on attorney/client privilege between himself and Otto and Otto's father?A) No, because he was never hired by Otto or his father.B) No, the privilege is a client privilege, not the attorney's privilege.C) Yes, assuming that Otto or his dad paid Bernie something for the consul.D) Yes, because the presence of a third party did not negate the privilege.
    Bernie may invoke the attorney/client privilege even though a 3rd party, Otto's father, was present. This doe not serve to extinguish the privilege. Attorney/client privilege protects any privileged communication made by a client while seeking the professional services of the attorney, even if it is before the lawyer has decided whether or not to take the case. Privileged communications are those that by definition are not disclosed to a 3rd party. If made in the presence of a stranger, the communication cannot be privilegeD) However, here we have a 3rd person whose presence is reasonable and understandable (a father helping his minor child), allowing the privilege to continue.A is wrong. Attorney/client privilege does not depend on whether the client hires the attorney or the attorney agrees to take the case, but simply that the client was seeking that attorney's professional advice.B is wrong since the person who was the attorney can, in fact, claim the privilege on behalf of the client, absent any evidence to the contrary.C is wrong since the privilege exists, absent any sort of payment for services.
  82. Complete this sentence: A federal district court judge, except in an extraordinary situation, . . .A) may not question a witness who has been sworn and who is an actual party tothe case or controversy at issue.B) may not direct questions to a sworn witness regarding matters that are theultimate issue of the case.C) may not question a non-expert lay witness in the trial.D)is free to question any of the sworn witnesses in any of the above situations.
    Under the federal rules judges are free to call witnesses and may further question already called witnesses as they see fit.The only boundary a judge may not cross is showing partisanship or preferential treatment to one side.
  83. Arnold is on trial for assault with a deadly weapon. At trial, Arnold's attorney puts Dr. Dray on the stanD) Dr. Dray is a scientist who conducts a yearly exit poll at the local ice cream shop of the entire town. Through his polling, he has determined that Arnold is a peaceful guy who likes bunnies and little kids. He contends that the community as a whole is "a bunch of whoring and drugging degenerates" but that Arnold is nothing like that according to his poll. On cross-examination, the prosecutor asks Dr. Dray, "Didn't you file a bogus insurance claim last year for a broken window?' Arnold's attorney objects. Should the court compel Dr. Dray to answer the question?A) Yes, but only if Dr. Dray was convicted of filing a bogus insurance claim in a court of law.B) Yes, it is a question relating to the truthfulness of the witness.C) No, introduction of specific instances of conduct are not permissible.D) No, because the question does not go to a relevant character trait.
    Here, we have a matter that serves to prove or disprove the credibility of a witness. Therefore, it is relevant and should be admitteD) Specific "bad acts" that relate to deceit and serve to show that a witness cannot be trusted, are clearly probative of the witness's honesty and integrity. A false insurance claim is clearly such an act. The question should be admitted and Dr. Dray compelled to answer.A is incorrect since it is perfectly fine to inquire into a witness's prior "bad acts" for impeachment purposes, even if the witness was never convicteD)C is incorrect. The federal rules allow for inquiry into specific acts of misconduct, as long as they are related to the honesty of the witness. Event then, the inquiry is at the court's discretion and the use of extrinsic evidence is not permissible.D is incorrect because the question relates to truthfulness, and a witness's credibility is always relevant.
  84. Daisy is riding the elevator in a department store when the elevator takes off at a high speed, throwing around everyone inside it. She tries to hold on but gets thrown into the wheelchair of the passenger beside her. Daisy suffers a great pain in her chest as her lung collapses from the impalement. The department store calls an ambulance. As they load Daisy onto a stretcher, she hears a bystander say, "You know, when I was here yesterday, I ran into my friend, Saily She was complaining to the owner that elevator was acting strange. I guess she was right." Daisy is treated for her injury, but is destined for a year of rehaB) She sues, seeking to introduce the testimony of the bystander. How should the court rule?A) Admissible, as a statement against interest.B) Admissible, as evidence that is relevant relating to the owner's knowledge of a broken elevator and his failure to have it fixeD)C) Inadmissible, unless Sally can be found to testify.D) Inadmissible, as hearsay not within any exception.
    Hearsay is a statement, other than one made by the declarant while testifying at the trial or hearing, offered in evidence to prove the truth of the matter asserteD) Here, we have a negligence case where knowledge of a danger is in issue. The statement of the third party customer, therefore, is applicable as an out-of-court statement offered to show that the store's owner had notice. In light of this, it should be admitted as an exception to the general hearsay rule.A is wrong as it misstates an exception to the hearsay rule. If Daisy's attorney was trying to introduce that the elevator was broken and used the statement to prove this as fact, then it is not a statement by a customer against that customers' interest, making this exception incorrect.C is incorrect since, as set forth in the correct answer, we are dealing with admissible nonhearsay. The fact that the declarant, Sally, is unavailable is meaningless. Unavailability only applies to certain hearsay exceptions like statements against interest. Here, the statement is only being used to show that the owner had notice.D is wrong as the customer's statement is not being used to prove the truth of the matter asserted (that the elevator was broken), but rather simply to prove that the owner was on notice of a problem.
  85. Trista sues Tzeitel, an acquaintance, due to a letter that was received at Trista's place of business that was libelous. It is alleged that the author of the letter was Tzeitel, but the authenticity of the letter is in dispute during the jury trial. Trista's attorney questions Trista's employer, Mrs. Kim, about the letter. Mrs. Kim responds, "Yes, I did receive a letter from Tzeitel about TristA) I lost it, but I can tell you exactly what it saiD)" The only way that Mrs. Kim can, in fact, testify as to what the letter said is if:A) A proper foundation as to Mrs. Kim's good character, honesty and veracity is set forth by Trista's attorney.B) The judge agrees with Mrs. Kim and finds that the original letter is, in fact, lost.C) The jury believes that the original letter is lost and cannot be introduced into evidence.D) Mrs. Kim has a good enough memory to recall the exact contents of the letter without a need look to any other document.
    Here, we are dealing with a best evidence question, so a party must either produce the original document or come up with a valid reason for not doing so. Loss of the original document is deemed a valid reason for failing to produce it. The status of the letter as lost however, is a question to be addressed and determined by the court, not the jury, making Answer B correct and Answer C wrong.A is wrong and a totally bogus rule of law. There is no need to lay a foundation of truthfulness in order to admit this witness.D is wrong. There is no rule of law requiring a witness to recall the exact contents of a document without needing to reference another document.
  86. Assume that we are dealing with a breach of contract case and the defendant's attorney is cross-examining a witness for the plaintiff. Which of the following questions would be found to be improper by the court?A) "Didn't your previous boss fire you when he learned that you reprogrammed the computer accounting system to deposit all the remainder cents into your personal account?"B) "Isn't it true that you are a creditor to the plaintiff who stands to make several thousand dollars if she wins this case?'C) "Isn't it true that you are currently seeking treatment for an addiction to painkillers that were provided to you by your maid?"D) "Did you not, Mr. Hancock, get yourself convicted in the state of New York on a forgery rap?"
    The federal rules will, in fact, allow cross-examination that is related to prior bad acts, if the court finds that the line of questioning is probative of truthfulness. In light of this, we can instantly rule out Question A as improper. It may be askeD)B is incorrect. A showing of a monetary interest which may result in a bias on the part of a witness is always relevant and admissible in a trial.D is incorrect. The federal rules do allow questions relating to prior crimes involving dishonesty if they are less than 10 years olD)Having ruled out all the other answers, the question relating to drug abuse, which may result in an inability of the witness to remember things form the past, is only marginally relevant to the matter at hand and will likely be found to be improper by the court.
  87. Assume that we are dealing with a contract case between Felipe and Uri in federal court. At dispute is the value of a house and its land called Cozy Cottage. Felipe calls Yolanda, a well know realtor in the area, to set a property price as an expert witness. The court finds Yolanda qualified as an expert witness. Yolanda has never been to Cozy Cottage so Felipe's attorney plans on asking her hypothetical questions based on the description of Cozy Cottage as set forth in the trial. Yolanda loves hearing herself talk and speaks for 3 straight hours. Felipe's attorney outright asks Yolanda, "So what is Cozy Cottage worth in your opinion?" Uri's attorney objects. How should the court rule on the objection?A) Sustained, an expert is never allowed to offer an opinion on the ultimate issue of a case.B) Sustained, when dealing with expert testimony an expert must disclose the basis for their testimony.C) Overruled, once a witness is qualified as an expert there is a presumption that the witness has a proper basis for any opinions offereD)D) Overruled, Uri's attorney is entitled to cross-examine Yolanda immediately.
    Under the federal rules a hypothetical question is unnecessary. Felipe's attorney is free to ask the expert opinion on valuation, but then must immediately allow Uri's attorney to cross-examine YolandA) In light of this, Answer B is clearly incorrect.A is wrong. Under the federal rules, the finding that an expert cannot testify as to the ultimate question at issue is no longer the case.C is one of those totally bogus pseudo-rules of law that are to be avoided at all costs on the test. Clearly it is wrong. Yolanda may be a qualified expert in real estate, but she has no basis to offer her opinion on everything from neurology to fashion.
  88. Yuliana and Jay help Candace unload boxes of her books onto a loading dock in front of her new apartment. Candace loves books and is using a forklift to carry her heavy collection. As she is driving the forklift, she collides with Kendra in her car. As they collide, the load of books tumbles to the ground and falls on Eve, who was walking in the roaD) Eve is hurt. A shopkeeper across the street sees all this happen and calls the police who arrive to take statements. Kendra sues Candace for negligence, claiming that Candace should not have been driving a forklift in the roaD) Kendra also asserts that she was driving in observance of all the traffic laws. Candace claims that Kendra really swerved into her and the forklift, because Kendra was distracted on her cell phone. All of the witnesses are available to testify at trial. Kendra's attorney calls the police officer to the stand to testify that, while Yuliana and Candace were standing together, Yuliana said, "Candace, you drove way too far into the oncoming traffic with the forklift," and that Candace stood there without saying a thing. How should the judge rule on this evidence?A) Admissible, since Yuliana is Candace's agent, as she was working for her and as an agent can, in fact, make a statement against interest B) Admissible, the silence of Candace is deemed an admission of guilt.C) Inadmissible, because Yuliana's statement and Candace's silent response are hearsay.D) Inadmissible, since Yuliana has yet to testify about this statement.
    In a situation like this, silence can be considered an admission. A reasonable person in the presence of a police officer taking a statement, would likely have spoken out against a statement of accusation like the one Yuliana made, if they were, in fact, not at fault.A is incorrect. There are no facts to indicate that we are dealing with an agency relationship. Furthermore, a statement of interest application would require that the declarant was not available, which is clearly not the case here.C is wrong. A court will find that the silence of a witness is deemed the adoption of the statement of another. Therefore, Yuliana's statement is considered an admission.D is wrong. There is no need to lay a proper foundation prior to the introduction of this evidence.
  89. Assume that we are dealing with a battery case. Tyson sues Mohammad for battery. Mohammad claims that he was attacked by Tyson for no apparent reason, and in self-defense hit him with the tire iron he was using to change his tire. Mrs. Lenox is called to the stanD) Mrs. Lenox lives in the house next to where Mohammad's car broke down. Mrs. Lenox will testify that on the night of the attack, she heard Mohammad and George talking and that George told Mohammad that Tyson had been flirting with his lover. George will also testify that he could see Mohammad get angry. Mohammad's attorney objects to the testimony of Mrs. Lenox. How should the court rule?A) Admissible, as this testimony serves to show provocation for a battery.B) Admissible, as a legally operative fact.C) Inadmissible, because it is hearsay.D) Inadmissible, because it is an opinion.
    The testimony of Mrs. Lenox is not being offered to prove that Tyson had been flirting with Mohammad's lover, but rather to prove that George's accusation had an effect on MohammaD) In light of this, we are not dealing with a hearsay statement and this statement can be admitteD) Therefore, Answer C is wrong.B is wrong. The concept of legally operative fact applies to situations where the words themselves are in issue. This is clearly inapplicable here.D is wrong. When dealing with witness opinion evidence, the evidence can be admitted if: it is based on the perception of the witness, helpful in determining a fact in issue, and not based an scientific, technical, or other specialized knowledge. In the present case, Mrs. Lenox's opinion testimony could be admitteD)
  90. Thornton is walking to class when he sees a red SUV blow through a red light, hit an elderly pedestrian knocking him unconscious, and continue on through the intersection. Thornton calls the police and gives a detailed description of wha he saw. The police capture Michaleen. They charge him with numerous counts, including hit-and-run and battery of the elderly pedestrian. Thornton testifies at trial, but Michaleen manages to get acquitteD) The elderly pedestrian then sues in a civil suit against Michaleen for damages relating to his injuries. Just before the civil suit, as luck would have it, Thornton is run over by a runaway truck full of hogs at the same exact intersection. The truck is driven by Wayne. An ambulance rushes Thornton to the hospital who with his last breath calls out, "Michaleen ran over that elderly man. I will never forget his face. He is guilty! Guilty, I tell your Thornton then dies. At trial, the elderly man's attorney offers the police report containing Thornton's description of the events and the driver. Michaleen's attorney objects. How should the court rule?A) The report is admissible as relevant and non-hearsay.B) Admissible, as this is a business record exception to hearsay.C) Inadmissible, because the report is hearsay not within any exception.D) Inadmissible, as violative of the Best Evidence Rule.
    Here, we have classic hearsay, namely an out-of-court statement being offered for its truth (the person who ran over the elderly pedestrian is Michaleen). In light of this, Answer A is incorrect.B is wrong. This does not fall within a business record exception to hearsay, as Thornton has no business obligation to provide facts to the police upon seeing the accident.D is incorrect. Under the Best Evidence Rule, the actual police report is the best evidence and therefore, that is the document that should be useD)
  91. Alita is riding up an escalator in Bloomstrom's, a department store, when the thing goes haywire, speeding up and slowing down violently. Alita falls down the escalator and crushes some of her ribs. Gilberto, a customer, rushes over to help her, calls an ambulance on his cell, and sits with Alita, trying to comfort her until help arrives. Gilberto has no medical training to speak of, but watches a lot of hospital dramas. Alita whispers to Gilberto, "Thanks for your help. I'm loosing my vision and fear that I'm suffering a blood clot in my brain caused by my fall." The ambulance arrives. They treat Alita and realize that there is no clot, but that she does have some serious nerve damage and broken ribs. Alita sues Bloomstrom's. Bloomstrom's response is, "This woman is bluffing. There is no way that she was that injureD)" At trial, Alita's attorney calls Gilberto to testify to the statement she made. Her statement is:A) Admissible to prove the extent of Alita's injuries after the accident, but before she had treatment.B) Inadmissible, since Alita never had a blood clot.C) Inadmissible, Gilberto is not a doctor.D) Inadmissible, here we have hearsay not within any exception.
    Here, we have a declaration of present body condition, which courts have found to be admissible as a hearsay exception, regardless to whom the declaration is made. Gilberto does not need to be a doctor in order to be a witness to the statement. Therefore, Alita's statement can be admitteD) In light of this, Answer D is clearly wrong.C is incorrect. As set forth in the correct answer, a declaration of present body condition can be admitted even if it was not made to a physician.B is incorrect. Alita's statement is admitted as a declaration relating to her pain at the time of the accident, not as proof that she had a blood clot affecting her brain.
  92. TIMELINE:1960: Hideaki buys life insurance from a door to door salesman, with his sister, Nara, as the beneficiary.1980: Hideaki goes hunting in Alaska for elk with a return date in 3 months. Nara goes to the airport at the end of those 3 months, but Hideaki never arrives on his scheduled flight. 2000: Hideaki has still never been heard from. Alaskan authorities have failed to find his body. He seems to have just disappeareD) Nara calls the insurance company with the facts, but they refuse to pay without evidence that Hideaki is actually deaD) Nara sues to collect the policy amount.The jurisdiction in which Nara sues has a statute on the books to address situations like this. It states that if a person has not been heard from in 8 years, they are presumed deaD) Which of the following is the most accurate statement?A) The jury is free to find that Hideaki is alive and still hunting somewhere in AlaskA)B) The jury can find that Hideaki is deaD)C) The judge must find that Hideaki is actually dead as a conclusive presumption and has no other choice.D) The jury must find that Hideaki is dead.
    The jury must find that Hideaki is dead as Nara has set forth all the required facts to prove this presumption - i.e., no one has heard from Hideaki in over 8 years. The opposing party, the insurance company, has not met its burden of providing that Hideaki is, in fact, alive by offering rebuttal evidence to the contrary. Answers A and B are, therefore, wrong.C is wrong. The insurance company can submit a rebuttal presumption, namely evidence to rebut the presumption that Hideaki is deaD) Answer C would only be correct if dealing with a conclusive presumption, as opposed to the rebuttal presumption at issue.
  93. Katina sues in probate court to contest the will of her mother, Mrs. Frangakis. Katina contests that at the time of the will's execution 10 years ago, Mrs. Frangakis was a drunk and incapable of making any sort of valid testamentary decisions. To support this claim, Katina seeks to introduce the testimony of Latifah, an attorney. Latifah will testify that 10 years and 6 months ago (6 months prior to the execution of the will) Mrs. Frangakis went to her to have a will made, but that she turned her away for being incapable of making any sort of coherent decisions due to alcohol impairment. The judge should rule this affidavit to be:A) AdmissibleB) Inadmissible, this violates the attorney/client privilege.C) Inadmissible, it is hearsay.D) Inadmissible, it is improper opinion evidence.
    Here, we have an affidavit that is clearly hearsay and there is nothing in the facts to indicate that it should be admitted as admissible evidence. In light of this, Answers A and B are clearly wrong. Answer B is wrong because the attorney/client privilege terminates upon the client's death. Once Mrs. Frangakis passed on, any privilege with Latifah terminateD) Additionally, the attorney/client privilege generally does not apply to observations made by the attorney about the client. "Communications received from a client" are the only things protected under the attorney client/privilege.D is incorrect. Based on the facts, a lay person would likely be allowed to testify as to their opinion since this opinion is: rationally based upon the lay person's perception; helpful to a determination of a fact in issue; and not based on scientific, technical, or other specialized knowledge. Meeting such requirements, Latifah's testimony as to the intoxicated state of Mrs. Frangakis is admissible.
  94. Ethel is injured in a car accident when her car is plowed into head on by Amir, driving one of his sports cars. Assume that we are dealing with the personal injury trial aspects and Ethel alleges that Amir was driving down the wrong side of the road, since he is British and forgot for a moment that he was driving in the US. Ethel also alleges that Amir was speeding, as he had his KPH and MPH confuseD) Amir denies all of this, aside from the fact that he is British. Ethel's attorney seeks to introduce evidence of Amir's reputation in the community as a crazy British daredevil, and for being genuinely reckless and irresponsible. Ethel's witness will also testify that Amir's nickname is "No Fear Amir" by his friends. This proposed testimony is likely to be found?A) Admissible, character can be proved by introducing reputation evidence.B) Inadmissible, if used to show that Amir was negligent in this situation.C) Inadmissible, because it is irrelevant.D) Admissible, if Amir's attorney opens the door by introducing evidence of Amir's reputation for carefulness and responsibility.
    In a civil case such as this one, character is only admissible as proof of conduct if character is directly in issue. That is not the case here, making Answers A and D incorrect.Note: In a criminal case the defense may introduce evidence of character, which in turn can be rebutteD)Evidence of character in the present fact pattern is clearly relevant to the court's determination of whether Amir is at fault, making Answer C incorrect.
  95. Viola is the owner of Spinnaker, an ocean-themed carnival ride. When a bolt on the ride snaps, Orsino's harness disconnects and he is pulled by the centrifugal force into the center of the ride. Unfortunately, his foot remained fastened to the ride and as a result was gruesomely ripped off when he fell. Viola's chief safety inspector puts together a report requested by her insurance company after the accident. Viola also forwards this report to her attorney. Viola's attorney seeks to introduce the report into evidence, as it says that the Spinnaker ride was in good condition at time of the accident. Orsino's attorney objects vehemently. The court will likely find the report to be:A) Admissible, as a business recorD)B) Admissible, as long as it is relied upon by a testifying expert witness.C) Inadmissible, since Viola's insurance company is an interested party in the litigation.D) Inadmissible, it is hearsay and not within an exception.
    Here, we have the perfect example of hearsay outside any exceptions. It is not a business record exception, as the report was not generated as a part of Viola's daily business routine, but at the insistence of her insurance company.B is incorrect. The mere fact that an expert relies on inadmissible evidence does not mean that the evidence is now admissible.C is incorrect. Interest of the insurance company results in a modification of the weight that is to be placed on the evidence, but it does not relate to the overall admissibility of the evidence as a whole.
  96. Dumaine owns a bakery, and is charged with murdering Hansel, his employee, by putting him in the baking oven with some muffins. The prosecution seeks to introduce the testimony of Oliver, another employee of Dumaine. Oliver will testify that he was using Dumaine's computer to buy some Canadian prescription drugs online, when he received an instant message from an unidentified person that said, "Hey Dumaine, I hear that Hansel is going to testify about all the illegal prescription drugs that you have been selling out of the bakery. What are you going to do about him?" Dumaine's attorney objects to the introduction of Oliver's testimony on the basis that it is irrelevant. How should the judge rule?A) Sustain the objection.B) Sustain the objection on the basis that any probative value the testimony may have is severely outweighed by the fear of unfair prejudice, or misleading the jury.C) Overrule the objection.D) Overrule the objection, but if the defense requests that the judge instruct the jury to ignore Oliver's testimony in relation to any acts of drug dealing by Dumaine, the judge must do so.
    Here, we have relevant evidence, as the evidenced offered helps establish a motive and in turn Dumaine's guilt. Evidence such as this is clearly relevant. Therefore, an objection by Dumaine's attorney based solely on relevance (as the question asks) should be overruleD) Once this is established, Answers A and B are clearly wrong.D is wrong. When dealing with a drug dealing question as presented here, the involvement of Dumaine remains relevant. Additionally, since the objection was based on grounds of relevance the limiting instruction of the judge is not requireD) Note: Had the objection been on hearsay grounds, the outcome may have been different. Here, however, as stated in the question, the objection is based on relevance grounds. ONLY answer what has been asked of you after careful reading.
  97. Clyde is charged with robbing a bank, but in the federal court case alleges that he was vacationing 5 states away in Wisconsin when the robbery occurreD) Clyde calls Bonnie as an alibi to testify that she was traveling on vacation with Clyde at the time in question. When questioned where she was during the robbery and with whom, Bonnie says, "Gee. / can't remember, but I have a pretty poor memory. I'm not even sure who I was out with yesterday." Bonnie does state that she vaguely recalls a weekend spent at the Grizzard Motel in Wisconsin, but she can't recall the exact date or who her companion was that evening. Clyde's attorney produces a letter written by Bonnie on Grizzard Motel stationary. He asks Bonnie to read the letter and try to answer the question again. The prosecution instantly objects. How should the court rule on the objection?A) Overruled, we are dealing with a past recollection recordeD)B) Overruled, but Bonnie cannot rely on what is written in the letter when she answers.C) Sustained, the letter is hearsay.D) Sustained, the letter has not been properly authenticated by Clyde's attorney.
    When dealing with a witness whose memory is incomplete, the defense attorney is free to refresh her memory with a prior writing, in this case Bonnie's letter on the motel stationary. When it comes time to testifying, however, the witness must testify from her present recollection and NOT on the prior writing. In other words, Bonnie can read the letter to refresh her memory and then testify, but she cannot testify based on what the letter says when question by an attorney.A is incorrect for misstating the use of a past recollection recorded exception to hearsay. This exception would only be applicable to a case where the party is seeking to introduce the writing into evidence. Here, the defense simply wants to use the writing to refresh the witness's memory.Both C and D are wrong since the defense is not seeking to introduce Bonnie's writing into evidence, but simply to refresh her memory of the weekend in question.
  98. Jeanette employs Catty and Kara at her Scrubby Bubbles & Bath store. Kara reports to Jeanette that she saw Catty stealing their "moonlight-scented" body scrub samples and putting them in her underwear. Livid, Jeanette fires Catty and brings a civil action against her for the value of the missing body scrub sample inventory. At the trial of the action, Jeanette's attorney calls Catty as an adverse witness, questioning her directly and asking if she stole the body scrub samples form the store. Catty refuses to testify, claiming a privilege against self-incrimination. The trial court should:A) Require that Catty disclose why body scrub samples were removed, such that the court can make a determination if Catty reasonably fears prosecution for a crime.B) Order Catty to answer, since the privilege against self-incrimination does not apply in civil proceedings.C) Sustain, since privilege against self-incrimination as outlined by the Fifth Amendment does not allow a compelled answer to questions that may tend to incriminate.D) Sustain only if it appears that Catty is likely to be criminally prosecuted by either the federal or state authorities.
    The Fifth Amendment of the United States Constitution provides that a witness cannot be compelled to testify against herself, thereby allowing Catty to refuse answering question that might incriminate her. This privilege applies to both civil and criminal proceedings, as long as the answer may expose a witness to criminal liability. This makes Answer B clearly incorrect.A is incorrect. When faced with potentially incriminating testimony, a witness is free to refuse to answer a question, regardless of whether the witness has a reasonable fear of potential prosecution.D is incorrect. The basis for applying the privilege against self-incrimination is not based on the likelihood of an actual criminal prosecution against the witness. Any testimony that could potentially open up a witness to criminal liability is enough to allow application of the privilege.
  99. Bear is a mechanic at a dealership who often test drives the cars that he fixes to be sure that everything works. On day, after fixing a LeDorien, Bear gets into a huge accident while speeding and showing off for some girls. The accident is with Boyd, who failed to stop at a red light. Boyd is thrown out of his car and lands in a pen of pigs. His car is totaleD) Bear realizes that Boyd is really hurt and runs to the pig pen to help him. Bear apologizes profusely saying, 7 did a rush job on this LeDorian and didn't fix the brakes as well as I usually do. I am SO sorry." After the accident, Bear readjusts the brakes. Boyd brings an action against the dealership where Bear works for damages. Assume that Boyd's attorney introduces evidence that Bear readjusted the brakes after the accident. Bear's attorney seeks to introduces a line of testimony regarding Bear's excellent driving history. This evidence is:A) Inadmissible, since it is character evidence.B) Inadmissible, because it is self-serving.C) Admissible, because it is character evidence.D) Admissible, because it is habit evidence.
    Here, the driving record is being admitted as character evidence. Generally speaking, the introduction of character evidence in a civil case is not permitted, as the small probative value is highly outweighed by the potential for distracting a jury. Introducing character evidence to infer that a person would generally act in conformity with their character is generally not alloweD) Attempting to illustrate that Bear has a good driving record and, therefore, likely acted in conformity with that good driving record is impermissible.B is incorrect. A court will not preclude the introduction of evidence simply because it is self-serving. Think about it. Just about any evidence a party offers is likely to be found self-serving. Why would a party introduce negative evidence against themselves?C is wrong as it is based on a belief that character evidence is generally allowed, which is clearly not the case in relation to civil cases in light of the correct answer. Note: Character evidence could be permissible in a situation where Bear's character was at issue (defamation, for example) but this is not the situation in the present case.D is incorrect since this is not habit evidence, which generally shows a person's response to a set of circumstances. Again, the attorney was attempting to introduced character evidence NOT habit evidence. This evidence does not show a response based on an ordinary set of circumstances the person typically encounters - i.e., habit evidence.
  100. Foxy runs into Randy with her SUV, leaving Randy with several injuries. The police arrive and require Foxy to take a breathalyzer test. She is found to be drunk and is citeD) She goes to court and is convicteD) Foxy gets the maximum penalty: 90 days in jail and her license revokeD) Randy sues seeking compensation for his neck injuries. Randy's attorney calls Wiley, a witness, and asks Wiley to describe Foxy's condition at the time of the accident. He describes Foxy's slurred speech, lack of balance and notes "Who is she kidding? She was totally drunk off her ass/" Foxy's attorney objects to this statement. How should the court rule?A) Sustained, because this is an opinion.B) Sustained, unless Wiley is an expert on alcoholic intoxication in human beings.C) Overruled, because this is proper opinion testimony.D) Overruled, because this is a present sense impression.
    Here, we have proper opinion testimony of a witness. Generally speaking, opinions are NOT admissible, but they are permissible in a situation where: 1) it is rationally based on the perception of the witness; and 2) it is helpful to a clear understanding of the witness's testimony or to the determination of a fact in issue. Wiley personally observed Foxy and his opinion that she was intoxicated benefits Wiley's testimony, since saying she appeared intoxicated is clearly saying that her speech was slurred, her gait was off and her eyes were bloodshot etC) . .A is incorrect. A lay opinion regarding a person's alleged intoxication can, in fact, be admissible.B is incorrect. Expert testimony is only necessary when dealing with a highly technical subject matter that is not easily understooD) A determination that a person is outwardly intoxicated does not require this level of expertise and can easily be determined by a lay person.D is incorrect. There is no hearsay situation here, yet Answer D states a hearsay exception. Hearsay is an out-of-court statement offered to prove the truth of the matter asserteD) Wiley is not testifying to an out-of-court statement, but rather to his opinion of Foxy's appearance at the time of the accident. Since there is no hearsay, there is no need for a hearsay exception.
  101. Ducky is driving a truck for ConvoyTrucking, when he runs over Kris and injures him. Kris sues, alleging that Ducky was drunk at the time of the accident and that ConvoyTrucking was negligent in hiring Ducky, who had a history of drunk driving. Kris's attorney calls the president of ConvoyTrucking as a hostile witness. The attorney attempts to get her to admit that after the accident she hired a professional urine-screening company to test all potential future company drivers for drugs and alcohol, and that she also set up a drug counseling program for existing employees. Will this be permitted by the court?A) Yes, to establish that the president of ConvoyTrucking was initially negligent in hiring Ducky and allowing him to operate a company truck.B) Yes, to show that the president of ConvoyTrucking knew of the need to better screen her employees.C) No, as such a situation would result in other tort-feasors never taking remedial measures when faced with a situation such as this.D) No, because it is not a proper form of impeachment.
    Under the federal rules, evidence showing remedial measures is not admissible to show negligence, as a public policy exists allowing people to make repairs or take other remedial measures after an accident for the benefit of public safety. Thus, A and B are incorrect.D is wrong as Kris's attorney is not trying to impeach the president of ConvoyTrucking, but rather trying to use her actions as proof of negligence.
  102. Steve is charged with killing Russell by throwing him into a furnace on October 9th. Steve's attorney calls Mike to testify to Steve's alibi. Mike states that he was with Steve in India on a trip to visit the Dalai Llama the week that the murder occurreD) On cross-examination, the prosecutor asks Mike, "Hey Mike, didn't you acquit Steve while serving on a jury trying him for another criminal charge?' Assume that the judge sustained an objection. This is likely because:A) The question is leading.B) The question goes beyond the scope of direct examination.C) The probative value of the answer would be substantially outweighed by its tendency to misleaD)D) A proper foundation was not laiD)
    Note: A juror cannot be a witness in a case in which they are sitting. By definition a juror is deemed an incompetent witness. Note, however, that this only relates to cases in which they are sitting on the jury.Here, Mike is not a juror in this particular case. Therefore, the only way that the evidence could be excluded is if the judge determines that the probative value of the evidence would be substantially outweighed by its tendency to misleaD)A is wrong since we are on cross-examination, wherein a leading question is permitteD)B is wrong since the question, here, relates to a potential bias in Mike toward Steve. Therefore, we cannot exclude this question on a bias of scope of directexamination.D is wrong for being another bogus answer that is illogical. A foundation relates to the introduction of physical evidence, not testimony.
  103. Bill and his son, Theo, are driving home from a baseball game when Tony crashes into them. This occurs at an intersection where all junctions have a stop sign. Bill and Theo sue Tony. At trial, Bill's attorney calls Theo to testify that while sitting in the front seat of his dad's car he saw Tony's big blue van, and saw that Tony never even slowed, let alone stop, for the intersection. Further assume that Tony calls Samantha as a witness. Samantha will testify that she was at the intersection waiting to cross the street at the time of the accident. While the ambulance was packing everyone up to go to the hospital, Tony offered her $100 to testify that he had not run the stop sign. Will Samantha's testimony be permitted?A) No, it is hearsay not within any exception.B) No, it is not relevant to the issue of negligence.C) Yes, it is relevant and not hearsay.D) Yes, it is a declaration against interest by Tony.
    Here, we have an admission by conduct, as the attempted bribery can be viewed as an admission to guilt. An admission is never declared hearsay; therefore, it is allowed to come in as an admission. In light of this, Answer A is clearly wrong.B is wrong since this is evidence that helps to establish that the defendant actually believed that he had done something wrong to cause the accident.D is wrong on the basis that an admission is never found to be hearsay. There is no need to bring this declaration in as an exception to the hearsay rule. Additionally, a declaration against interest is only applicable in a situation where the declarant is unavailable. Samantha is clearly available, making Answer D wrong for yet another reason.
  104. GlamDiva is injured when her blinged-out tour bus is T-boned by a truck filled with bowling balls. It was GlamDiva's bus driver that ran a stop sign and caused the accident. Once the bus stopped rolling, David, a fellow passenger, called out, "Damn man, we had to be doing at least 80! This bus driver was flying!!! Is everyone okay?" GlamDiva sues Van Cool (the tour bus rental company) for her injuries. She sues on the theory of respondent superior and negligent hiring. GlamDiva's attorney seeks to introduce evidence that one year before the accident, but 5 years after the bus driver in question was hired, Van Cool instituted rigorous driving tests for all of their drivers. Since the bus driver was hired before the testing program was instated, Van Cool only checked to see that he had a bus license when they hired him and nothing more. In reality, he had 3 revoked licenses and a history of running stop signs. Can GlamDiva's attorney introduce the evidence of the new employment background checks?A) No, because it is irrelevant.B) No, because it is evidence of remedial measures.C) Yes, because it is evidence of Van Cool's negligence.D) Yes, because it is evidence that the bus driver was incompetent.
    This evidence tends to indicate that Van Cool was not, in fact, acting in a non-negligent manner when they hired the bus driver in question. Therefore, Answer A is clearly wrong.B is not a good answer. In the interest of public safety, courts generally will not allow subsequent remedial actions by a company to be introduced into evidence. Here, however, Van Cool's actions in question happened prior to the accident.D is wrong. It does not illustrate the bus driver's incompetence, but rather shows Van Cool's negligence in not properly screening and testing all of their drivers.
  105. Choy is a minor and, with his parents, sues Cristos for $100,000 in injuries that Choy sustained when Cristos ran over him one evening while delivering pizzas and rushing around to get them delivered in less than 3 hours. Choy was knocked to the sidewalk from his bike and rendered unconscious. Cristos claims that it was not he who hit Choy. Aside from damages, the real issue is whether, in fact, it was actually Cristos who hit Choy. Cristos' attorney asks Cristos, 7s it possible that Choy mistook your car for the car of someone else? After all, the kid did hit his head pretty badly" This question is:A) Objectionable, the answer would be hearsay.B) Objectionable, Cristos' answer would be an opinion.C) Unobjectionable, because the answer is relevant to the question of who hit Choy.D) Unobjectionable, if a proper foundation has been laiD)
    Here, we have a question of a lay person's opinion. Generally, such an opinion is permissible if it is rationally based on the perception of the witness, helpful in understanding the witness's testimony relating to a fact at issue and is not based on scientific or expert knowledge. Here, Cristos' lay opinion meets none of these requirements and is, therefore, inadmissible.A is wrong because the question asked does not require an answer that is an out-of-court declaration.C is wrong. Although relevant, the answer would still be improper. D is wrong because the form of the question is improper.
  106. Tracy dies without a will, leaving her only son, Meachum, as her heir. During probate, Nick, Tracy's "personal assistant," makes a claim for the house and produces a deed in which the property was allegedly transferred to him 2 months before Tracy died of a stroke, which was suffered while in the heat of passion. Meachum disputes the claim, arguing that the signature on the deed is fake. Assume that this all takes place in front of a jury. Tracy's son Meachum states that he has seen his mother's signature each month for his entire life, as Tracy typically sent him an allowance check, and that the signature on the deed is clearly not Tracy's. The judge should rule this testimony:A) Inadmissible, because Meachum is not deemed a handwriting expert with specialized skills in recognition.B) Inadmissible. Since Meachum holds a stake in the outcome of the case, one cannot trust his opinion.C) Admissible, since Meachum clearly knows what his mother's signature looks like.D) Admissible, since Meachum is disputing the validity of the signature on the document and not the validity of the document itself.
    Note: Any lay witness that knows the signature of a party may testify as to the validity of that party's signature, even a lay witness who holds a stake in the proceedings. In light of this, Answer A is clearly wrong.B is wrong. It addresses the belief the jury may have in the testimony's truth, i.e., the weight of the testimony, but fails to address the admissibility of the testimony.D is wrong. Meachum may testify in support of or against the genuineness of the signature.
  107. Kathy is on trial for killing her boyfriend Lee by feeding him into a wood chipper and directing the exhaust of the chipper into a salmon stock pond where the fish could eat Lee's body and grow into strong steelheads. Kathy claims that Lee jumped into the chipper to commit suicide. Kathy's attorney calls Phil who works at CallBeforeYouJump, a suicide prevention clinic, to testify. Phil alleges that on numerous occasions Lee went into the clinic mentioning that he was going to kill himself. The judge should rule the testimony to be:A) Admissible, because the statement was made in "contemplation" of Lee's death.B) Admissible, since it shows intent in Lee to commit suicide.C) Inadmissible, because it violates the psychiatrist/patient privilege.D) Inadmissible, because there is no evidence that Lee came into the clinic on the day that Lee was killed.
    We clearly have testimony that is hearsay and ordinarily would not be allowed into evidence. However, this evidence falls within the state of mind exception.A is incorrect because a "dying declaration" must be made while in fear of "impending" death. Clearly, this was not the case.C is incorrect. We have no facts indicating that Phil is a psychiatrist or a psychologist, so there is no clear doctor/patient confidentiality. Note that Phil does not actually need to be a psychiatric professional in order to testify; Lee only needs to have thought that he was one.D is wrong. Lee's overall state of mind roughly around the time of the incident is relevant enough. Phil's knowledge need not be confined to the exact day of the incident in order to be relevant.
  108. Lynnell, Elaine, Chris and Zeke are driving to Graceland to meet Elvis in a diner when out of nowhere they are run off the road and over a cliff by a King Delivery Co. truck, driven by Aaron. Though Aaron was driving the truck, he had a buddy named Buddy along for the ride. Lynnell, Elaine, Chris and Zeke sue King Delivery Co. and Aaron. Their attorneys allege that the King Delivery Co. was negligent since Aaron had been awake for 36 hours straight on a trip from California to GracelanD) The King Delivery Co. answers the complaint by claiming that Aaron was actually high on psychedelic drugs and cocaine when the accident occurreD) They seek to introduce the testimony of Officer Priscilla, a Tennessee Highway Patrol officer who overheard Aaron say just after the accident, "Awe damn, dude. Buddy, this was probably my fault since I'm all drugged up on blow and shroomsr Assume that the court takes official notice that "shrooms" are psychedelic mushrooms and "blow" is cocaine. The plaintiffs' attorneys seeks to introduce the statement of Buddy, who just before the accident told Aaron, "Aaron, you better get some rest You're swerving all over the place and nodding off. You know what the boss says about driving tireD) Pull over before you kill someoneF' The judge should rule the statement admissible only if A) The statement was made immediately after the accident and was made under oath.B) Buddy cannot be found to testify to the statementC) The plaintiffs' attorneys produce a record of three tickets issued in the last 6 months by the Califonia Highway Patrol who repeatedly pulled Aaron over for reckless driving in the same truck.D) The plaintiffs' attorneys first lay the proper foundation by establishing that Buddy used to work for the King Delivery Co. and that Buddy's statement was within the scope of his knowledge since he used to be an employee.
    The federal rules state that when dealing with the out-of-court statement of an agent, the statement can come in, as long as it was made during the existence of a relationship between agent and principal, and concerns the scope of that agent's duties or knowledge. If this is the case, Buddy's statement would be an admission of a party-opponent and, therefore, fall within an exception to the hearsay rule. Answers A and C state requirements for admissibility that have no basis in law. Answer B is also incorrect as a statement admission is not based on whether the declarant of that statement is available to testify.
  109. A bank is robbed on a Monday morning by a man dressed as a leprechaun. The man is 3 foot 6 inches tall. On Tuesday, the police get a tip from Jon saying that Jake, a person also of short stature, has treated the entire condo complex to drinks at a local bar, bragging that he just struck it rich and flashing a briefcase full of $100 bills. The police go to the condo, knock and Jake answers. The police ask about the rolls of hundreds and Jake states that an old friend paid off an old debt regarding a bet they haD) The police leave, only to be called back by the custodian who states that he found a green leprechaun costume and a fake beard in the recycle bin. The police return and arrest Jake. After obtaining a search warrant, they search his condo only to find nothing. On cross-examination, Jake's attorney asks Jon, "Didn't Jake's ex-wife pay you $5,000 to call the police with the tip about the $100s? And didn't she also pay you another $5,000 to testify against Jake today?' The prosecutor objects. The objection should be:A) Overruled, because the question allows Jon to go ahead and affirm or deny the accusation.B) Overruled, since this is a valid way to impeach a witness.C) Sustained, because the question addresses a collateral issue.D) Sustained, because it is a leading question.
    Here, we have a valid way to show bias in a witness and to subsequently impeach him. Therefore, it is allowable.A is related to the requirements for introducing a prior inconsistent statement. Here, we have no facts indicating that this is a prior inconsistent statement and in light of this, the answer is clearly wrong.C is incorrect. When dealing with the credibility or potential bias of a witness, we are not dealing with an issue deemed collateral to the question at hanD)D is wrong as the attorney is conducting a cross-examination, during which a leading question is permissible.
  110. Chad sues Clarita for injuries he suffered while working for him as a cesspool cleaner. Clarita files a cross-complaint. It states that Chad was negligent when he cleaned the cesspool with gasoline and continued to smoke his cigarette while entering the cesspool, thereby, causing the explosion that both damaged the drain and injured ChaD) Chad denies these allegations. Clarita's attorney calls Lulu to testify. She declares that while in the cesspool with Chad, she heard Tom an employee who died in the explosion say, "Chad! For Pete's sake put out that damn cigarette! Are you nuts?! This place is gonna blowf The judge should rule that this testimony is:A) Inadmissible, because it is hearsay not within any exceptions.B) Inadmissible, because it violates the Dead Man Statute.C) Admissible, because it is an excited utterance.D) Admissible, because it is a dying declaration.
    Here, we have a clearly excited utterance by Tom which falls within an exception to the hearsay rules. In light of this, A is wrong as there is an applicable exception at work.B is wrong because this statute only limits the admissibility of testimony against a descendent. Here, we are faced with testimony that does not subject Tom or his estate (following his death) to any sort of liability. Therefore, the statement is permissible.D is incorrect. When working in a jurisdiction which allows dying declarations in civil cases, the declaration must be made at a time when the declarant (Tom) knows that imminent death is going to occur. Additionally, that declaration must concern the cause of the declarant's death.
  111. Sylvester and Rea collide with each other in their fancy sports cars. Rea is indicted for driving drunk, which has a maximum sentence of 2 years in jail. Blathe, who witnessed the entire accident, testifies in front of the grand jury. Rea pleads guilty and she is fined $500. After completion of the criminal proceedings, Sylvester sues Rea in a negligence suit seeking personal injury damages. Sylvester's attorney offers a properly authenticated record of Rea's drunk driving conviction. The record should be:A) Admitted, as proof of Rea's negligent character.B) Admitted, as proof of Rea's intoxication.C) Excluded, because the conviction was not the result of a trial.D) Excluded, because it is hearsay without exception.
    The record may be admitted into evidence to prove intoxication. The record of conviction is hearsay, but falls within the hearsay exception for records of felony convictions. Under the federal rules, judgments of felony convictions are admissible in both criminal and civil actions to prove any fact essential to the judgment, whether the judgment arose after trial or upon a plea of guilty.Note: A felony is any crime punishable by death or imprisonment for more than one year.A is incorrect. Because this is a civil case, evidence of character used to prove conduct is generally inadmissible, because such evidence creates a danger of prejudice and typically distracts the jury from the main issues.C is incorrect. A felony conviction judgment can be admitted, regardless of whether the conviction resulted from a trial or a guilty plea, as set forth in the correct answer.D is incorrect because the judgment falls within a hearsay rule exception for records of felony convictions.
  112. Strauss sues Morrow and Morrow's employer based on a personal injury claim in which Strauss is hit on the head by a bucket of paint that was dropped by Morrow while she was painting the Golden Strait Bridge. To prove that the paint was dropped by Morrow, Strauss's attorney introduces the now empty and dented paint can. The can has a sticker attached stating the paint color: "Strait Bridge golD)" Strauss's attorney also introduces evidence that the same brand of "Strait Bridge gold" paint was sold to Morrow and her employer for use in painting the bridge. Suppose a state OSHA officer interviews everyone connected with the accident, and during the interview Morrow's employer makes several damaging statements. These statements are recorded in the OSHA officer's report. From the following answers, which one is NOT a foundational fact that Strauss's attorney will need to establish in order to introduce the OSHA report as a past recollection recorded exception to the hearsay rule?A) The report was written while the interview was fresh in the memory of the OSHA officer.B) The report accurately records what was said by Morrow's employer.C) The report was written by the safety officer or adopted by him.D) The safety officer is unavailable and cannot be called as a witness at trial
    The past recollection recorded exception to the hearsay rule does not require unavailability of the declarant. Past recollection applies to a witness who has insufficient memory of an event to enable himself to testify fully and accurately, regardless of the fact that he may have revised (either intentionally or unintentionally) the writing while on the stanD) If this is the case, then the writing may be introduced into evidence following a proper foundation. The foundation requires proof that: 1) the witness at one time had personal knowledge of the facts recited in the writing; 2) the writing was made by or under the direction of the witness or has been adopted by him; 3) the writing was timely made when the matter was fresh in the mind of the witness; 4) the writing is accurate; and 5) the witness has insufficient recollection to testify fully and accurately. Answers A, B, and C each closely resemble one of the requirements set forth above. Answer A applies to condition 3, Answer B to condition 4 and Answer C to condition 2.
  113. Barnard sues NoChairLeftBehind, a moving company, after his couch falls off the back of their truck while he is following directly behind them on the highway in his car. Barnard's suit is based on the fact that NoChairLeftBehind was negligent by supplying its driver with a truck that had a broken loading liftgate which carried the couch. He also contends that the driver of the truck was additionally negligent in failing to properly secure the couch in place in lieu of the broken liftgate. NoChairLeftBehind has a company-wide policy that following an accident a driver must file a company report stating what happeneD) Barnard's attorney seeks to have this report introduced by NoChairLeftBehind and they comply. He then calls the driver as an adverse witness. The driver states that he did fill out the required report just after the accident and that it states the cause of the accident as the broken liftgate. Barnard's attorney seeks to introduce the report into evidence and NoChairLeftBehind's attorney objects. The report is:A) Admissible, only if the driver states that he cannot recall the details of the report.B) Inadmissible, since it was not part of the ordinary course of business for NoChairLeftBehinD)C) Inadmissible, as only factual business records can be admitted into evidence. An opinion, such as the accident report, is not fact.D) Admissible as nonhearsay.
    Here, we" have an admission by a party-opponent. It is also a statement by an agent concerning a matter within the scope of his agency, made during the existence of the employment relationship. Admissions such as this are considered nonhearsay and can, in fact, be admitted into evidence.A is wrong as this is a nonhearsay related admission that does not require the laying of a foundation in order to be admitteD)Note: Even if the report had not been made by a party in the case, it could still be admitted as a past recollection recorded, but the proper foundation would have to be laid in for it. Examples of this foundation include: 1) the witness at one time had personal knowledge of the facts recited in the writing; 2) the writing was made by the witness or under his direction, or was adopted by the witness; 3) the writing was timely made when the matter was fresh in the mind of the witness; 4) the writing is accurate; and 5) the witness is presently unable to remember the facts sufficiently to testify fully. Here, these requirements are not applicable to a past recollection recorded application, as the driver has not testified that the writing is accurate or that he has insufficient recollection. Therefore, the report would not be admitted as a past recollection recordeD)B is wrong as it implies that the evidence may only be admitted as a business recorD) Since the proper application of this evidence is as an admission as set forth in the correct answer, a businessrecord exception is unnecessary.C is also wrong in view of the correct answer, as we are dealing with an admission which does not require a foundation in order to be admitted into evidence.
  114. Cami sues Bruce for frauD) She seeks damages following the purchase of a charter fishing business from Bruce. Cami claims that she relied on Bruce's bookkeeping as to the value of the business based on assets and inventory (frozen bait). Cami claims that the bookkeeping turned out to be fraudulent. At trial, Cami's attorney introduces the written contract, without objection from defense, which reads, "HolyMackeral is a solvent company with inventory and assets valued at $1 million." The contract further includes some boilerplate terms like "no other additional or contradictory representations have been made by the seller to the buyer beyond those written above, etC).." Note that Bruce is a lawyer, while Cami is not. As part of his defense, Bruce calls Dobbs as a financial expert to the stanD) Bruce asks Dobbs about his PhD work in finance, his training, work experience and the universities he graduated from. On cross, Cami's attorney asks Dobbs, "Didn't you fail out of Leelow University after a year?" Dobbs says "No." Cami's attorney seeks to call the Leelow University's dean of admissions with a copy of Dobbs' old transcript. Bruce objects. What is the best reasoning to follow when dealing with the question of whether to admit the transcript into evidence?A) The plaintiff is trying to impeach the witness on a collateral matter, which cannot be proved by extrinsic evidence.B) The judge should admit the transcript and decide himself if Dobbs is a qualified expert.C) The qualifications of an expert witness are determined by the jury. The judge should admit the transcript and tell the jury to ignore Dobbs' testimony if they don't find him an expert D) The scope of Dobbs' degrees is irrelevant to his expert testimony and the transcript cannot be admitteD)
    A judge is responsible for determining the qualifications of an expert witness and typically must consider any relevant evidence relating to an expert's qualifications. Therefore, the transcript should be evaluateD)A is incorrect. Dobbs claiming a PhD is not a collateral matter, as a witness's competence and credibility is always relevant. When trying to determine whether something is collateral, simply ask whether the evidence would be admissible absent the contrary assertion by the witness. If yes, then you can't exclude the impeachment evidence as collateral matter. Cami's attorney can easily present evidence that Dobbs is unqualified, regardless of whether Dobbs claims to have a PhD, making the transcript admissible.C is wrong. The jury is not responsible for determining whether Dobbs is an expert witness. Once the jury is allowed to hear Dobbs' testimony, it is too late (for reasons of prejudice) to instruct them to ignore his testimony as an expert witness. That is why an expert witness must be qualified by the judge.D is incorrect. Any matter relating to the competence of a witness is relevant and Dobbs' credibility as a witness can be undermined simply by showing that he never got a degree from Leelow University.
  115. Minka is charged with bank robbery. During her prosecution, it is well established that the robber left the bank wearing a dog mask and got in a getaway car. Bill, Gill and Dan all witness the getaway. Bill gives testimony in a grand jury trail that states, 7 got the plate number of the getaway car. It's 1234567" Because Bill cannot be found for trial, the prosecutor seeks to introduce the transcript of Bill's testimony from the grand jury proceedings. This testimony is:A) Admissible to rehabilitate Bill if Bill is impeached by Dan during the trial.B) Admissible nonhearsayC) Hearsay, but it can be admitted under the former testimony exception.D) Inadmissible hearsay
    Hearsay is a statement, other than one made by the declarant while testifying at the trial or hearing, offered in evidence to prove the truth of the matter asserteD) Here Bill's statement was not made at Minka's trial and is, in fact, offered for truth. Minka's attorney had no opportunity to cross-examine Bill during the grand jury trial. Bill's statement is inadmissible hearsay since it does not fall within any exception to the hearsay rules.A is wrong since we have inadmissible hearsay, and this hearsay can not be admitted, even if for the purpose of rehabilitation.B is wrong since we are dealing with hearsay. There is a semi-applicable potential exception for a prior consistent statement made by a witness and its subsequent introduction: To use the statement for the purpose of rebutting a charge of fabrication by the witness. This exception, however, is inapplicable to the present situation.C is wrong. The former testimony exception to the hearsay rule requires that the party against whom the former testimony is offered (Minka), must have had a chance to directly examine, cross-examine, or examine the witness on redirect. A grand jury case does not award her this chance.
  116. Yuliana and Jay help Candace unload boxes of her books onto a loading dock in front of her new apartment. Candace loves books and is using a forklift to carry her heavy collection. As she is driving the forklift, she collides with Kendra in her car. As they collide, the load of books tumbles to the ground and falls on Eve, who was walking in the roaD) Eve is hurt. A shopkeeper across the street sees all this happen and calls the police who arrive to take statements. Kendra sues Candace for negligence, claiming that Candace should not have been driving a forklift in the roaD) Kendra also asserts that she was driving in observance of all the traffic laws. Candace claims that Kendra actually swerved into her and the forklift, because Kendra was distracted on her cell phone. All of the witnesses are available to testify at trial. Candace's attorney seeks to introduce testimony from the police officer who took the shopkeeper's statement which says, "That woman in the car was swerving all over the place and ran into the girl with the forklift." The judge should rule the testimony:A) Inadmissible, the Best Evidence Rule would require the production of the police officer's original report containing the shopkeeper's statement.B) Inadmissible, because it is hearsay.C) The oral testimony of the police office is inadmissible, but the police report could be admitteD)D) Admissible, under the past recollection record exception, as applied to the police officer.
    Here we have hearsay of the shopkeeper, and it remains hearsay, even though it is recorded in the police report.Both A and C are wrong as the statement of shopkeeper remains hearsay even though the police report may, in fact, qualify under some exceptionD is wrong for 2 reasons. First, we have no evidence that the police officer is testifying from the report and secondly, even if he were doing so, this does not change the fact that it is hearsay.
  117. Clyde is charged with robbing a bank, but in the federal court case alleges that he was vacationing 5 states away in Wisconsin when the robbery occurreD) Clyde calls Bonnie as an alibi to testify that she was traveling on vacation with Clyde at the time in question. When questioned where she was during the robbery and with whom, Bonnie says, "Yes, / definitely know that I was at the Grizzard Motel the weekend of the robbery, because that is the same weekend as my birthday and I was there to celebrate. The thing is, there's no way I was there with a guy as ugly as Clyde." The prosecution seeks to introduce testimony from Clyde's ex-wife, Blanche, as evidence. Blanche will testify that, prior to the robbery, she found Clyde trying panty hose on over his head in the bathroom, as well as a blueprint of the bank in Clyde's sock drawer. Clyde's attorney vehemently objects. How should the court rule?A) Sustained, because spousal privilege belongs to the party-spouse. B) Sustained, because Clyde's actions were in reliance on his spousal relations with Blanche.C) Overruled, the federal rules do not recognize any privileges relating to compelled or prevented testimony.D) Overruled, Blanche is free to testify as she chooses.
    In dealing with a criminal case in federal court, a spouse is free to testify as they please against their spouse, regardless of whether their spouse seeks to prevent it. They cannot, however, be FORCED to testify against their spouse. Inlight of this, Answer A is clearly incorrect.B does, in fact, state one of the elements relating to spousal privilege. In the present case, however, Blanche's information was not acquired due to her marital relationship with Clyde, but rather information she found out on her own. Spousal privilege does not apply to a scenario such as this.C is incorrect. The federal rules do not explicitly state which privileges exist. They allow the common law to govern the existence and enforcement of various privileges as recognized by US courts.
  118. Choy is a minor and, with his parents, sues Cristos for $100,000 in injuries that Choy sustained when Cristos ran over him one evening while delivering pizzas and rushing around to get them delivered in less than 3 hours. Choy was knocked to the sidewalk from his bike and rendered unconscious. Cristos claims that it was not he who hit Choy. Aside from damages, the real issue is whether, in fact, it was actually Cristos who hit Choy. Cristos seeks to testify that each night he delivers pizzas, he takes the same road back to the pizza parlor where he works. He further seeks to testify that this is NOT the road where Choy was hit. The judge should find the offer to testify:A) Objectionable, as this would be a self-serving declaration.B) Objectionable, as irrelevant.C) Unobjectionable, as this would be habit evidence.D) Unobjectionable, if it is offered as impeachment evidence.
    Habit evidence can be introduced to the present case by Cristos as long as he does, in fact, always take the same road back to the pizza parlor where he works.A is wrong because "self-serving" evidence is not a valid objection. Just about any evidence that a defendant offers is in some way or another self-serving.B is wrong because the evidence may, in fact, show that it was not Cristos who ran over Choy's head like a crushed watermelon. Therefore, it is clearly relevant.D is wrong. Habit evidence is admissible to show habit as well as to impeach. It is not for impeachment purposes only.
  119. Yuliana and Jay help Candace unload boxes of her books onto a loading dock in front of her new apartment. Candace loves books and is using a forklift to carry her heavy collection. As she is driving the forklift, she collides with Kendra in her car. As they collide, the load of books tumbles to the ground and falls on Eve, who was walking in the roaD) Eve is hurt. A shopkeeper across the street sees all this happen and calls the police who arrive to take statements. Kendra sues Candace for negligence, claiming that Candace should not have been driving a forklift in the roaD) Kendra also asserts that she was driving in observance of all the traffic laws. Candace claims that Kendra actually swerved into her and the forklift, because Kendra was distracted on her cell phone. All of the witnesses are available to testify at trial. Kendra's attorney seeks to introduce a statement that Yuliana made to the police officer that said, "Jay, Candace and I had few cocktails before unloading things, and we probably overdid it a bit." The trial judge should rule that this evidence is:A) Inadmissible, since it is relevant to an issue presented in the currently pending trial.B) Inadmissible, it is a hearsay declaration.C) Admissible, it is an admission.D) Admissible, as a statement against interest.The statement is clearly hearsay and is inadmissible.
    A is wrong as this statement does, in fact, tend to show a reason why the present accident occurreD)C is wrong because Yuliana is not a party in this case and the statement can not be attributed to Candace.D is wrong. In order to apply the statement against interest exception, the witness must be unavailable. Here, we have no indication that Yuliana is unavailable, as the facts clearly state that all the witnesses are available to testify at trial.
  120. Chalky and Jimmy are driving their scooters down the street when Chalky manages to collide with Jimmy who was distracteD) Part of the reason for the accident is because Jimmy was sloshed, after a night of drinking and partying with his friend's banD) Jimmy is arrested for possession of cocaine, which carries a maximum penalty of 4 years in the can. Steph saw the entire event transpire. She testifies at a grand jury trial, during which Jimmy eventually pleads guilty and is fined $5,000 with no jail time. One week later, Chalky decides to sue Jimmy to recover for his damaged scooter that was his pride and joy. Chalky's attorney, being sly, decides to introduce the record of Jimmy's drug possession suit. How should the court rule on the record?A) Excluded, since Jimmy's conviction was not the product of a trial, but rather a plea in a grand jury investigationB) Excluded, because it is hearsay not within any exception.C) The court will admit the record on the basis that it serves as character evidence useful in the suit at hand D) The court will admit the record, as it establishes proof of Jimmy's drugging ways.
    This record can, in fact, come in to prove that Jimmy was high on drugs, since the record falls within the prior felony conviction exception for hearsay. If this was not a felony conviction and applicable exception, this record would be inadmissible hearsay. The federal rules allow any felony convictions to be admitted in to both criminal and civil trials in order to prove a fact in issue. It doesn't mater that there was a plea and no trial for this exception to apply.A is wrong in light of the correct answer, which sets forth that any felony conviction, regardless of whether it was by way of a trial or by way of a plea bargain, can be admitteD)B is also wrong in light of correct answer, since we are dealing with a hearsay exception.C is incorrect. A court will rarely allow evidence of character admitted in to civil case since this has the end effect of being generally prejudicial to the defendant.
  121. Frank is charged with forging 10 checks from his employer, which he made out to himself. The total amount of the allegedly forged checks is $1.6 million. The prosecutor is a young lawyer and this is his first case. He has several witnesses ready to testify about the alleged forgeries. From the following choices which person should the prosecutor NOT present the handwriting material to, since their evaluation of the writing sample will be impermissible?A) A police officer, who arrested Frank when he was trying to use one of the checks to buy 712 porn magazines. The officer also has a copy of Frank's driver's license as a writing sample.B) Frank's former employee, who knew Frank's signature from his paycheck.C) An expert witness known for signature analysis, who has looked at both Frank's handwriting and the alleged forgery on the check.D) The jury in the case, following a handwriting sample of Frank from his driver's license test 10 years ago, as well as a copy of the alleged forgery.
    Handwriting analysis such as this requires the following elements:1) personal familiarity with Frank's writing, 2) a basis for forming an evaluative opinion of the handwriting based on specialized knowledge, or 3) a particular fact-finding function, such as that assigned to the jury.B is wrong. Element 1 allows a former employee with prior knowledge of Frank's signature to testifyC and D are incorrect because an expert witness or the jury fall within elements 2 and 3, and are free to evaluate the writing. The police officer is neither a trier of fact nor an expert, so element 2 and 3 do not apply.
  122. The MetroFree is a free newspaper read by thousands of people a day on their way to work. On Wednesday, they run a front page article stating that, "Walker is a draft-dodging pinko communist who likes to kick old ladies for fun." Walker notices this on his way to work one morning and flips his lid, since he is clearly not a communist nor an anti-geriatriC) He fears that his membership in the Fun Republicans will get revoked and instantly sues for defamation. At trial, Walker seeks to put Condi Risotto on the stand to offer testimony that he once donated $1 million to AIDS research. MetroFree's lawyers object to this testimony. How should the court rule?A) Inadmissible, the testimony offered by Risotto is unrelated to any material issue in the caseB) Admissible, Walker's character is an issue and he is free to rebuke claims of his bad character.C) Inadmissible, character cannot be proven by a specific instance of good character.D) Admissible, as evidence of Walker's good character.
    Risotto's testimony has no relation to any material issue of the case. At best her testimony shows Walker to be a philanthropic kind of guy, but she makes no mention of whether he is a communist or anti-geriatric, etC) . . As this testimony lacks probative value, a judge would not let it in.C misstates the actual law. When character is directly in issue, a court WILL allow specific instances of good character to show that a person likely acted in conformity with his character.D is incorrect. A court will only allow character evidence to come in when a person's character is in issue. In this case any evidence must, therefore, relate to Walker's treatment of old ladies, political background, or service recorD) Unfortunately for Walker, this testimony has no relation to any of these and will not be permitteD) In a similar line of reasoning, Answer B is incorrect as the evidence offered is simply not relevant to the matter at hanD) In light of this, a judge would not let it in.
  123. Barack and Hillary are pilots, who manage to accidentally crash into each other head on. Their planes go spiraling into a factory and crash through the roof. Luckily for both of them, the factory builds trampolines and after bounding several times they finally come to rest, shaken but not harmeD) Dean, an employee, comes out after using the bathroom and sees the two crashed planes laying there. Dean used to work for Flighty Airlines before they fired him for drinking the little rum bottles on the job and eating too many peanuts. Dean notices that the throttle in Barack's plane was pushed all the way forward, while the one in Hillary's plane was halfway forward, an indication that the plane flown by Barack was going faster. Hillary's attorney calls Dean to the stand to testify about the position of the throttle in Barack's plane. Barack's attorney objects. Should the testimony be admitted?A) No, unless Hillary's attorney established that Dean is a qualified expert in attesting to such matters.B) Yes, since it is Dean's personal observation.C) No, unless Hillary's attorney shows that another witness can corroborate what Dean saw.D) Yes, it can be admitted as Dean's opinion.
    The only way this testimony can be admitted, is if Dean is established to be an expert in accident reconstruction such as this. We have no facts to indicate that Dean is any sort of expert, and only expert technical knowledge can provide an estimation of speed based on the position of a plane's throttle. Being a former employee with a drinking problem and a peanut fetish probably isn't enough to establish Dean as an expert in the fielD) In light of this, his testimony probably would not be admitteD)C is incorrect. There is no rule of law requiring a witness to be present to corroborate Dean's testimony or finding, regardless of whether he is an expert in the field or simply offering opinion testimony.D is incorrect. Absent a showing that Dean is an expert (which we have established is not the case) all he is offering here is his opinion. In the present case, in light of the need for expert testimony, Dean's opinion testimony is not permissible. Answer B is wrong in a similar vein. Testimony based on personal observation, absent a showing that the person is an expert, will not be allowed to come in.
  124. Anna and Boris contract for 1,000 Venetian mirrors. After receiving the shipment, Boris is angry that the mirrors look "a little too Rococo" and sues based on a dispute in the contract. Anna's attorney decides to call Barry to testify regarding his personal knowledge of the agreement between Anna and Boris. Anna's attorney also intends to call Gwen to further testify that Barry is the most honest person she knows, has never lied once, and is an honest chap. Boris's attorney objects to this proposed testimony and the court sustains the objection. What is the court's reason for preventing Anna's attorney from calling Gwen to testify regarding Barry's truthfulness?A) The credibility of Barry was never in issue.B) Any testimony by Gwen is inadmissible as hearsay not within an exception.C) Opinion testimony regarding honesty is not allowed by Gwen in relation to Barry.D) In a civil case such as this, a court will generally not allow reputation testimony to be admitted
    In the present fact pattern, Barry's credibility is never in issue. Therefore, the court has no reason to allowing Gwen's testimony to further support a question that is not in issue. Testifying to a person's credibility is only applicable when credibility of the witness is in question (i.e., via an impeachment attempt). Here, that is not the case at all, so there is no need for additional testimony.B recalls the defense of hearsay: namely, an out-of-court statement offered in evidence to prove the truth of the matter asserteD) Here, there is no testimony by Gwen to an out-of-court statement, but rather testimony to her opinion of Barry's character for truthfulness.C is wrong since a court will allow the credibility of a witness (either the good credibility or the bad) to be attacked based upon opinion evidence relating to the witness's honesty. Here, Barry's honesty was not attacked so there is no need to offer the supporting testimony of Gwen.D is incorrect for misstating the law. The reputation of a witness for truthfulness can, in fact, be admitted in to a civil case if used to impeach. RECALL that it isn't reputation, but rather character that generally cannot be introduced into a civil case, unless that character itself is in issue.
Author
Anonymous
ID
16037
Card Set
Evidence Final for 2Ls
Description
Multiple Choice questions for Evidence - Helpful for Evidence final/the bar exam.
Updated